Vous êtes sur la page 1sur 64

C

sciences
physiques et chimiques
A
P
J.-P.DURANDEAU • J.-L.BERDUCOU • J.-C.LARRIEU-LACOSTE •
C. MAZEYRIE • C. RAYNAL • J.-C. TRILLAUD

livre du professeur
Table des matières

Électricité
  1 Le circuit électrique . . . . . . . . . . . . . . . . . . . . . . . . . . . . . . . . . . . . . . . . . . . . . . . . . . . . . . . . . . . . . . . . . . . . . 5
  2 L’intensité du courant électrique . . . . . . . . . . . . . . . . . . . . . . . . . . . . . . . . . . . . . . . . . . . . . . . . . . . . . . . . . 8
  3 La tension électrique . . . . . . . . . . . . . . . . . . . . . . . . . . . . . . . . . . . . . . . . . . . . . . . . . . . . . . . . . . . . . . . . . . . 11
  4 La tension sinusoïdale . . . . . . . . . . . . . . . . . . . . . . . . . . . . . . . . . . . . . . . . . . . . . . . . . . . . . . . . . . . . . . . . . 14
  5 Puissance et énergie électriques . . . . . . . . . . . . . . . . . . . . . . . . . . . . . . . . . . . . . . . . . . . . . . . . . . . . . . . . . . . . 17
  6 Prévention des risques électriques . . . . . . . . . . . . . . . . . . . . . . . . . . . . . . . . . . . . . . . . . . . . . . . . . . . . . . 20

Chimie
  7 Prévention des risques chimiques . . . . . . . . . . . . . . . . . . . . . . . . . . . . . . . . . . . . . . . . . . . . . . . . . . . . . . . 23
  8 Éléments chimiques et atomes . . . . . . . . . . . . . . . . . . . . . . . . . . . . . . . . . . . . . . . . . . . . . . . . . . . . . . . . . . 25
  9 Les molécules . . . . . . . . . . . . . . . . . . . . . . . . . . . . . . . . . . . . . . . . . . . . . . . . . . . . . . . . . . . . . . . . . . . . . . . . . 28
10 Les solutions ioniques . . . . . . . . . . . . . . . . . . . . . . . . . . . . . . . . . . . . . . . . . . . . . . . . . . . . . . . . . . . . . . . . . . . . . 32
11 Solutions et concentrations . . . . . . . . . . . . . . . . . . . . . . . . . . . . . . . . . . . . . . . . . . . . . . . . . . . . . . . . . . . . . 35
12 Solution acide, neutre ou basique . . . . . . . . . . . . . . . . . . . . . . . . . . . . . . . . . . . . . . . . . . . . . . . . . . . . . . . 38
13 Techniques d’analyse et de dosage . . . . . . . . . . . . . . . . . . . . . . . . . . . . . . . . . . . . . . . . . . . . . . . . . . . . . . 41

Mécanique et Thermométrie
14 Le mouvement rectiligne . . . . . . . . . . . . . . . . . . . . . . . . . . . . . . . . . . . . . . . . . . . . . . . . . . . . . . . . . . . . . . . 44
15 Le mouvement circulaire . . . . . . . . . . . . . . . . . . . . . . . . . . . . . . . . . . . . . . . . . . . . . . . . . . . . . . . . . . . . . . . 47
16 Actions mécaniques - Forces . . . . . . . . . . . . . . . . . . . . . . . . . . . . . . . . . . . . . . . . . . . . . . . . . . . . . . . . . . . . 50
17 Équilibre d’un solide soumis à deux forces . . . . . . . . . . . . . . . . . . . . . . . . . . . . . . . . . . . . . . . . . . . . . . . 53
18 Moment d’une force - Moment d’un couple de forces . . . . . . . . . . . . . . . . . . . . . . . . . . . . . . . . . . . . . 57
19 Les sons . . . . . . . . . . . . . . . . . . . . . . . . . . . . . . . . . . . . . . . . . . . . . . . . . . . . . . . . . . . . . . . . . . . . . . . . . . . . . . 60
20 Température et changement d’état . . . . . . . . . . . . . . . . . . . . . . . . . . . . . . . . . . . . . . . . . . . . . . . . . . . . . . 62

Couverture : npeg.fr
Maquette et mise en page : Nicolas Balbo et Catherine Vielcanet

© HACHETTE LIVRE 2010, 43, quai de Grenelle, 75905 Paris Cedex 15


ISBN 978-2-01-180787-8
www.hachette-education.com

Tous droits de traduction, de reproduction et d’adaptation réservés pour tous pays.


Le Code de la propriété intellectuelle n’autorisant, aux termes de l’article L. 122-4 et L. 122-5, d’une part, que les « copies ou
reproductions strictement réservées à l’usage privé du copiste et non destinées à une utilisation collective », et, d’autre part,
que « les analyses et courtes citations » dans un but d’exemple et d’illustration, « toute représentation ou reproduction intégrale
ou partielle, faite sans le consentement de l’auteur ou de ses ayants cause, est illicite ».
Cette représentation ou reproduction, par quelque procédé que ce soit, sans autorisation de l’éditeur ou du centre français de
l’exploitation du droit de copie (20, rue des Grands-Augustins, 75006 Paris), constituerait donc une contrefaçon sanctionnée
par les Articles 425 et suivants du Code pénal.
Sciences physiques et chimiques

Les connaissances abordées dans cette partie du référentiel de certification sont réparties
en unités communes à tous les CAP et en unités spécifiques attribuées en fonction des
secteurs professionnels.

Dans les unités communes, la formation dispensée participe au développement des


savoirs fondamentaux et à l’appropriation de méthodes. Elle doit permettre de développer
des attitudes comme le sens de l’observation, la curiosité et l’ouverture d’esprit, l’esprit
critique et l’intérêt pour les progrès scientifiques et technologiques. Elle doit faciliter un
changement de voie de formation, voire une poursuite d’études, mais aussi l’adaptation à
l’évolution de la profession. L’unité commune Sécurité (S) est une unité transversale, qui
doit être intégrée aux différentes unités de chaque secteur professionnel.

Les unités spécifiques apportent aux élèves des méthodes et des connaissances dans les
champs particuliers des sciences physiques et chimiques afin de faciliter l’appropriation des
formations professionnelles. Les unités spécifiques retenues pour un secteur professionnel
donné sont celles dont l’apport est particulièrement important pour la formation
professionnelle correspondante. Le professeur de sciences physiques et chimiques est
encouragé à développer l’enseignement des unités spécifiques et à choisir des situations
d’évaluation en relation étroite avec ses collègues de l’enseignement professionnel.

Unités communes
Sécurité (S) : prévention des risques chimiques et électriques
Chimie 1 (Ch. 1) : structure et propriétés de la matière
Chimie 2 (Ch. 2) : acidité, basicité ; pH
Mécanique 1 (Mé. 1) : cinématique
Acoustique (Ac.) : ondes sonores
Électricité (El.) : régime continu, régime sinusoïdal monophasé, puissance et énergie

Unités spécifiques
Chimie 3 (Ch. 3) : techniques d’analyse et de dosage
Mécanique 2 (Mé. 2) : équilibre d’un solide soumis à deux forces
Mécanique 3 (Mé. 3) : moment d’un couple
Thermique (Th.) : thermométrie

3
Programme et référentiel de certification
en fonction des spécialités de CAP

Le tableau ci-dessous indique, pour chaque groupement de secteur professionnel, les


unités repérées par des croix (X) qui font partie du programme de formation et qui
peuvent faire l’objet d’une évaluation.

• Groupement A : CAP des actuels secteurs 1, 2 et 3


• Groupement B : CAP des actuels secteurs 4 et 5
• Groupement C : CAP des actuels secteurs 6 et 7

Groupement A Groupement B Groupement C

Productique -
Maintenance
Bâtiment - Travaux Tertiaires - Services
publics Hygiène - Santé Hôtellerie
Électricité - Chimie et procédés Alimentation -
Électronique Restauration
Audiovisuel
Industries graphiques

Sécurité X X X

Chimie 1 X X X

Chimie 2 X X X

Chimie 3 X

Mécanique 1 X X X

Mécanique 2 X X

Mécanique 3 X

Acoustique X X X

Électricité X X X

Thermique X X

4
1 Le circuit électrique
Les activités
■■Activité 1 Le courant électrique a-t-il un sens ?
• Matériel 
Un générateur ou une pile plate avec deux pinces crocodiles.
Un moteur.
Un interrupteur.
Des fils de connexion.

• Réponse aux questions


1. Les bornes du générateur sont marquées et colorées : rouge (+) et noire (-). Pour une pile plate, la
borne (+) est la lame la plus courte.
2. Le moteur tourne seulement si l’interrupteur est fermé : le générateur est nécessaire pour faire
circuler le courant dans le circuit.
3. Le sens de rotation du moteur dépend du sens de son branchement sur le générateur.

■■Activité 2 Comment brancher plusieurs dipôles ?


• Matériel 
Un générateur ou une pile plate avec deux pinces crocodiles.
Deux lampes identiques adaptées au générateur.
Un interrupteur.
De la paille de fer.
Des fils de connexion.

• Réponse aux questions


Expérience 1
1. Le circuit en série comprend une seule boucle, le circuit en dérivation en comprend deux.
2. Si une lampe du circuit en série est hors service, le circuit ne fonctionne plus.
Expérience 2
1. Les lampes n’éclairent pas. Seule la boucle ABCDEF ne comprenant aucune lampe est parcourue
par le courant pendant un bref instant.
2. La paille de fer a servi de fusible et a grillé.

CHAPITRE 1 - Le circuit électrique • 5


Les exercices
Je sais 7. Schématiser
a. b.
1. Q.C.M.
1 : A et B 2 : B 3 : B et C
4 : A 5 : A et C 6 : C.

2. Brancher les fils


a. Dipôles en dérivation :

8. Décrire le schéma
a. Deux lampes sont branchées en dérivation (on dit
aussi « en parallèle ») aux bornes d’un générateur.
b. Un générateur, un interrupteur, une lampe et une
b. Dipôles en série : résistance sont branchés en série.
c. Un générateur, un moteur et un groupement de
deux lampes en parallèle sont branchés en série.

J’applique
9. Schématiser un circuit

3. Repérer les bornes d’une pile


Borne + Borne – Borne +

– +

Borne – 10. Feux tricolores


Sur la photo, le bonhomme rouge est éclairé : le cir-
4. À la fermeture cuit électrique qui l’alimente est constitué d’un grand
Circuit a  : le courant circule dans la branche de la nombre de DEL rouges. Lorsqu’une de ces DEL est
DEL 2 : elle brille. La DEL 1 est éteinte. hors d’usage, le bonhomme rouge reste éclairé, car
Circuit b  : le courant circule dans la branche de la les autres DEL continuent à fonctionner : elles sont
DEL 1 : elle brille. La DEL 2 est éteinte donc branchées en dérivation. Il en est de même des
DEL vertes.
5. Corriger les affirmations
a. Les deux lampes L3 et L4 sont montées en dériva- 11. En marche
tion. Le schéma a. est correct, car il faut appuyer simulta-
b. Exact. nément sur les deux poussoirs pour fermer le circuit.
c. Le courant circule dans le sens des aiguilles d’une
12. Expérience avec des isolants
montre : il sort de la borne (+) du générateur.
et des conducteurs
6. Vérifier le montage a. Isolants : matière plastique, papier, verre.
Le circuit photographié est un circuit en série  : le b. Conducteurs : aluminium, acier, graphite, cuivre.
moteur ne doit pas être dessiné en dérivation, mais c. Autre isolant : le bois.
en série dans le schéma du circuit. d. Autre conducteur : l’or.

6 • CHAPITRE 1 - Le circuit électrique


Le document

■■Va-et-vient et télérupteur
• Réponse aux questions disjoncteur
ou
1. Schéma : coupe-circuit
– en petits pointillés : télérupteur
le circuit de commande ; boutons-
– en larges pointillés : DISJ A1 A2 poussoirs
le circuit des lampes. 1
0
2. Schéma ci-contre.
1 2
10A
3. Le circuit de commande n’est parcouru
par un courant que pendant la durée phase
d’actionnement d’un bouton-poussoir. neutre
4. Le disjoncteur divisionnaire situé
en amont, ici de 10 ampères, protège
le système d’une surintensité.

lampes

PRÉPARER LE CCF

Le montage tête-bêche
Le tableau ci-dessous propose un barème d’évaluation pour aider à la notation de chaque élève.
Des fiches analogues, directement exploitables par l’enseignant, peuvent être téléchargées
sur le site de Hachette Éducation.

Vérifications Exploitation des résultats expérimentaux

1. Vérification des branchements Justifier pourquoi la DEL verte brille .......................... 1 pt


(résistance et DEL verte)............................. 2,5 pts
Schéma du montage et sens du courant
2. Vérification du branchement (2 × 0,5 pt) .................................................................................... 1 pt
(DEL rouge) ........................................................ 2,5 pts
Justifier pourquoi aucune DEL ne brille
3. Vérification de la réalisation (2 × 0,5 pt) ..................................................................................... 1 pt
du pont AB................................................................. 1 pt
Remise en état du poste de travail ............. 1 pt

TOTAL ....................................................................... 7 pts TOTAL .............................................................................. 3 pts

CHAPITRE 1 - Le circuit électrique • 7


2 L’intensité du courant électrique
Les activités
■■Activité 1 Comment mesurer la valeur de l’intensité du courant ?
• Matériel 
Une lampe adaptée.
Une pile.
Un ampèremètre.
Des pinces crocodiles.
Des cordons.

• Réponse aux questions


1. Le sélecteur est placé sur la zone continue, sur le calibre 10 A.
2. On utilise les bornes 10 A et COM.
3. L’intensité du courant traversant la lampe est de 0,21 A.

■■Activité 2 Quelles sont les lois des intensités dans les circuits ?
Expérience 1. Comment se répartit l’intensité du courant dans un circuit série ?

• Matériel 
Un générateur.
Un ampèremètre.
Une lampe.
Un petit moteur.
Des cordons.

• Réponse aux questions


1. On utilise la zone intensité-courant continu ; les bornes COM et mA ; le calibre 200 mA.
2. On a la relation : IA = IB = IC.

Expérience 2. Comment se répartit l’intensité dans un circuit avec dérivation ?

• Matériel 
Un générateur.
Un moteur et une lampe adaptés.
Trois ampèremètres.
Des cordons.

• Réponse à la question
On a la relation : I = I1 + I2.

8 • CHAPITRE 2 - L’intensité du courant électrique


Les exercices
Je sais 8. Mesure d’intensité
a. Sens du courant dans le circuit :
1. Q.C.M.
1 : B ; 2 : A ; 3 : A ; 4 : B et C ; 5 : A ; 6 : C.
I
2. Convertir
En ampère :
740 mA = 0,74 A ; 74 mA = 0,074 A ; 0,74 kA = 740 A ;
7,4 kA = 7 400 A. M A
En milliampère :
0,04 A = 40 mA ; 0,40 A = 400 mA ; 4,0 A = 4 000 mA. I
I
3. Lecture d’intensité
I1 = 5,43 A. b. Lors du branchement de l’ampèremètre, les pola-
I2 = 5,43 mA. rités ne sont pas respectées.
I3 = 5,43 mA. c. Le moteur est traversé par un courant d’intensité
4. Le bon branchement 15,8 mA. Il en est de même pour la DEL.
Dans les schémas a. et c. l’ampèremètre mesure l’in- d. On a choisi le calibre immédiatement supérieur à
tensité dans le moteur. 15,8 mA, soit 20 mA.

5. Circuit série 9. Courant principal


a. On applique la loi d’unicité de l’intensité dans un a. Le schéma complété :
circuit série. COM
b. L’ampèremètre A2 indique 3,52 A. A
IL IM
6. Circuit dérivation I
a. Schéma du circuit : M

b. Le sens du courant sur le schéma.


c. I = IL + IM = 0,250 + 0,350 = 0,600 A
A
10. Trouver le branchement
a. On peut écrire I1 = I2 + I3.
A b. Schéma représentant ce circuit :

I3 L3
A

b. On applique la loi d’additivité des intensités dans I2 L2


un circuit dérivation.
L1 I1
On a :
I = I1 + I2 = 53,8 + 34,2 = 88,0 mA.

c. La lampe L1 est en série avec le générateur  ; les


lampes L2 et L3 sont montées en dérivation.
J’applique
11. La multiprise
7. Courant principal et courants dérivés
a. I = 7 + 2 + 0,7 + 0,3 + 1,5 = 11,5 A.
a. I1 = I2 + I3 = 2,5 + 3,4 = 5,9 A
b. En faisant fonctionner tous les appareils ensem-
b. I4 part du nœud B et I4 = I6 + I5 = 7 + 2,2 = 9,2 A.
ble, Chloé va créer une surcharge et le coupe-circuit
(fusible ou disjoncteur) de 10 A ouvrira le circuit pour
protéger l’installation.

CHAPITRE 2 - L’intensité du courant électrique • 9


Le DOCUMENT

■■L’intensité des courants dans un scooter


• Réponse aux questions
1. En série, la mise hors service d’une lampe entraînerait l’extinction des autres.
2. Schéma :
L1

L2

3. Le générateur est en court-circuit.


4. L’intensité du courant qui sort de la batterie est : I = 5 + 0,5 = 5,5 A.

PRÉPARER LE CCF

Utiliser un ampèremètre
Le tableau ci-dessous propose un barème d’évaluation pour aider à la notation de chaque élève.
Des fiches analogues, directement exploitables par l’enseignant, peuvent être téléchargées
sur le site de Hachette Éducation.

Vérifications Exploitation des résultats expérimentaux

1. Vérification du choix de la zone Convertir l’intensité en mA ............................................... 1 pt


(Courant continu - A)........................................... 1 pt
Pour mesurer un courant de 0,015 A soit 15 mA,
2. Vérification du branchement de
on utilise le calibre immédiatement supérieur
l’ampèremètre (calibre, bornes
soit 20 mA par exemple ...................................................... 2 pts
et polarités) ............................................................ 3 pts

3. Vérification du choix du calibre


adapté........................................................................... 1 pt
Lecture de l’intensité du courant. ............... 1 pt

4. Remise en état du poste de travail


(sélecteur sur 0) ...................................................... 1 pt

TOTAL ....................................................................... 7 pts TOTAL .............................................................................. 3 pts

10 • CHAPITRE 2 - L’intensité du courant électrique


3 Tension électrique
Les activités
■■Activité 1 Comment mesurer la tension aux bornes d’un générateur ?
• Matériel
Une pile.
Une lampe adaptée.
Un interrupteur.
Un voltmètre.
Deux pinces crocodiles. V
Des cordons.

• Réponse aux questions


1. Ci-contre schéma du circuit.
2. Le voltmètre est branché en dérivation aux bornes de la pile.
Le sélecteur est sur la zone voltmètre continu. Le calibre choisi
est immédiatement supérieur à la tension mesurée, ici 20 V.
3. La tension aux bornes de la pile est de 4,79 V.

■■Activité 2 Loi des tensions


Expérience 1. Comment se répartissent les tensions dans un circuit en série ?

• Matériel
Un générateur.
Deux lampes.
Un voltmètre.
Un interrupteur.
Des cordons.

• Réponse à la question
On peut écrire la relation U = U1 + U2.
Expérience 2. Comment se répartissent les tensions dans un circuit avec dérivation ?

* Réponse à la question
On a la relation : U = U1 = U2.

Les exercices
Je sais COM
V
1. Q.C.M.
1 : C ; 2 : C ; 3 : B ; 4 : C ; 5 : B ; 6 : B.

2. Brancher un voltmètre.
a. Schéma ci-contre. M
b. Le multimètre est branché en dérivation ; il mesure
la tension aux bornes du moteur.
c. Voir schéma.
+ –

CHAPITRE 3 - Tension électrique • 11


3. Convertir 10. Identifier un circuit
Tension Autre a. Schéma du circuit :
Objets
en volt unité
Un réseau Haute Tension 50 000 V 50 kV
Un moteur de TGV 1 100 V 1,1 kV
Un signal du cerveau 0,025 V 25 mV L2 L3
Une cellule photovoltaïque 0,520 V 520 mV
(photopile)

4. Ouvert / fermé L1
a. UL = 0 lorsque K est ouvert.
b. UL = 12 V lorsque K est fermé.
5. Montage avec dérivation
a. Schéma ci-contre.
b. La tension aux
G b. L1 est branchée en dérivation.
bornes de L2 est de L1 c. On a U1 = U2 + U3 = 10 + 2 = 12 V.
12,16 V. La tension aux bornes du générateur est de 12 V.
c. La tension aux bor- L2 11. Batterie de 12 V
nes du générateur est Une batterie neuve et une batterie usagée peuvent
également de 12,16 V. avoir sensiblement la même tension à vide (f.e.m.)
V donc cette mesure ne renseigne pas sur l’état de la
6. Montage en série batterie.
a. On utilise la loi d’additivité des tensions d’une 12. Julie, Aurélie ou Agnès
branche : U = UL + UM . Lorsque Julie branche les deux cordons aux bornes
b. U = 2,1 + 4,2 = 6,3 V. du voltmètre, leurs deux extrémités sont soumises à
une tension nulle et leur contact accidentel n’a pas
J’applique de conséquence. Par contre, Aurélie dispose d’une
tension aux extrémités des cordons et leur contact
7. Deux circuits
accidentel provoquerait un court-circuit.
Pour le moteur et la lampe en série, on a : U = 12 V.
Agnès qui pense que les solutions sont équivalentes
Pour le moteur et la lampe en dérivation, on a : U = 6 V.
doit considérer qu’avec des cordons de sécurité, le
8. Résistance de protection contact accidentel des extrémités des cordons n’est
a. U2 = U – U1 = 4,5 – 1,9 = 2,6 V. pas possible.
b. Si on supprime la résistance, la DEL est en surten- La réponse dépend donc de la qualité des cordons
sion : elle est placée sous une tension de 4,5 V alors utilisés.
que sa tension nominale est : 1,9 V.
13. Un taille-haie
9. Quatre lampes Pour faire fonctionner le moteur du taille-haie, on ne
Avec une batterie de 6 V, on peut réaliser le montage dispose plus que d’une tension de :
suivant : 230 – 2 × 10 = 210 V.

14. Énergie rayonnante


a. La tension aux bornes du générateur est de :
500 × 24 = 12 000 mV soit 12 V.
b. La tension aux bornes de chaque lampe est alors
6V de :
12
= 6 V.
2

Chaque lampe fonctionne sous sa tension nominale


de 3 V.

12 • CHAPITRE 3 - Tension électrique


LE DOCUMENT

■■Adapter une lampe à un générateur


• Réponse aux questions
– On écartera L3 car sa tension nominale ne correspond pas à celle du secteur.
– Vu l’aire de la terrasse, on choisira L1 car, adaptée au secteur, elle brillera plus que L2 :
l’intensité nominale de L1 est 15 fois supérieure à celle de L2.

PRÉPARER LE CCF

Utiliser un voltmètre
Le tableau ci-dessous propose un barème d’évaluation pour aider à la notation de chaque élève.
Des fiches analogues, directement exploitables par l’enseignant, peuvent être téléchargées
sur le site de Hachette Éducation.

Vérifications Exploitation des résultats expérimentaux

1. Vérification du branchement Effectuer la mesure (préciser l’unité) . ........................ 1 pt


du voltmètre (zone, calibre, bornes Réponse aux questions
et polarités)............................................................. 3 pts – Le voltmètre se branche en dérivation
2. Vérification de l’adaptation avec la lampe ........................................................................ 2 pts
du calibre ................................................................. 2 pts – Pour mesurer la tension aux bornes de
3. Remise en état du poste de travail la pile de 1,5 V, on utilise le calibre de 2 V ............. 1 pt
(sélecteur sur 0)...................................................... 1 pt

TOTAL ....................................................................... 6 pts TOTAL .............................................................................. 4 pts

CHAPITRE 3 - Tension électrique • 13


4 La tension sinusoïdale
Les activités
■■Activité 1 Comment distinguer une tension alternative
d’une tension continue ?
• Matériel
Un oscilloscope.
Deux générateurs continu et alternatif 6 V.
Deux cordons.

• Réponse à la question
Avec la sortie continue, la tension reste constante. Avec la sortie alternative, la tension ne conserve
pas la même valeur au cours du temps.

■■Activité 2 Quelles sont les grandeurs caractérisant


une tension sinusoïdale ?
• Matériel
Un oscilloscope.
Un générateur de tension sinusoïdale.
Deux cordons.

• Réponse à la question
La tension sinusoïdale étudiée présente quatre grandeurs caractéristiques :
- la période en seconde,
- la valeur maximale ou amplitude en volt,
- la fréquence en hertz,
- la valeur efficace en volt.

Les exercices

Je sais 4. Période et valeur maximale


a. Voir graphique :
1. Q.C.M.
1 : B ; 2 : A et B ; 3 : A ; 4 : A et B ; 5 : A et C. U(V)
2. Aux bornes d’une batterie 2
a. Cette tension est continue. 1
b. U = 12,5 V. t(s)
c. Un voltmètre en mode continu. 0
50 100
3. Motif élémentaire et période –1
Un motif élémentaire est colorié en noir dans le
–2
graphique b.

b. T = 50 s. c. Umax = 2,3 V.


5. Fréquence
Il y a 5 périodes pour une durée de 1 s. La fréquence
est de 5 Hz.

14 • CHAPITRE 4 - La tension sinusoïdale


6. Tension efficace b. La DEL verte est allumée en permanence.
a. On mesure la tension efficace avec le voltmètre et
la tension maximale à l’aide de l’oscilloscope.
b. La tension efficace aux bornes du générateur est
de 3,01 V. R
c. Le sélecteur du voltmètre est en mode alternatif.

J’applique
7. Caractéristiques des tensions sinusoïdales
Tension
Période Fréquence Amplitude c. Les deux DEL sont allumées par intermittence.
efficace
T (s) f (Hz) Umax (V)
U (V)
0,020 50 17 12 G
0,01 100 325 230
R
8. Lire un oscillogramme
a. Durée de balayage :
5 ms/DIV ; pour une période, on a 4 divisions.
T = 4 × 5 = 20 ms soit 0,020 s.
b. Sensibilité verticale :
5 V/DIV ; la hauteur d’un pic est de 3 divisions.
La tension maximale est : 11. Plaque signalétique
Umax = 3 × 5 = 15 V a. La plaque signalétique comporte le symbole alter-
natif et l’indication de la fréquence nominale.
9. Quel réglage ?
b. f = 50 Hz ; U = 230 V.
a. Oscillogramme de la tension :
12. Tableur grapheur
a. Voir le graphique :

U(V)
8
6
4
2
t(s)
0
10 20 30 40 50 60 70 80 90
–2
–4
b. Cette tension est une tension continue. –6
c. La sensibilité verticale de l’oscilloscope est de :
–8
0,5 V/DIV.

10. Quel générateur ?


a. La DEL rouge est allumée en permanence. b. Cette tension est alternative et sinusoïdale.
c. La période T est de 80 s et la valeur maximale Umax
est de 6 V.
1 1
d. La fréquence f =  = = 0,0125 Hz ;
R T 80

la valeur efficace
U max 6
U =  = = 4,2 V.
2 2

CHAPITRE 4 - La tension sinusoïdale • 15


LE DOCUMENT

■■Les tensions alternatives d’un scooter


• Réponse aux questions
1. Pour qu’une tension alternative apparaisse aux bornes d’une bobine, il faut déplacer un aimant
devant une de ses faces.
2. C’est le rotor du volant magnétique qui transmet le mouvement et remplace la main sur
la manivelle de la chignole.
3. En faisant varier la fréquence de rotation des aimants, on fait varier la fréquence de la tension
alternative.
4. Lorsque le moteur est à l’arrêt, le rotor ne tourne plus et le courant électrique ne peut provenir
du volant magnétique pour alimenter les dispositifs électriques du scooter.

PRÉPARER LE CCF

Analyse de la tension de sortie d’un transformateur


Le tableau ci-dessous propose un barème d’évaluation pour aider à la notation de chaque élève.
Des fiches analogues, directement exploitables par l’enseignant, peuvent être téléchargées
sur le site de Hachette Éducation.

Vérifications Exploitation des résultats expérimentaux

1. Vérification des branchements Calcul de f et U......................................................................... 2 pts


(transformateur, oscilloscope).................... 2 pts
Réponse à la question 
2. Vérification des réglages de – La tension de sortie du transformateur
l’oscilloscope (balayage, sensibilité a la même forme et la même fréquence qu’à
verticale) .................................................................. 2 pts son entrée. La tension efficace de sortie n’est pas
égale à celle d’entrée (elle est multipliée par 4
3. Mesure de Umax et vérification
pour un transformateur 6V / 24V ................................... 1 pt
des mesures (Umax et T)................................... 2 pts

4. Remise en état du poste de travail ............. 1 pt

TOTAL ....................................................................... 7 pts TOTAL .............................................................................. 3 pts

16 • CHAPITRE 4 - La tension sinusoïdale


5 Puissance et énergie électriques
Les activités
■■Activité 1 Comment calculer la puissance électrique reçue
par une lampe ?
• Matériel
Un générateur.
Une lampe adaptée.
Un voltmètre.
Un ampèremètre.
Des cordons.

• Réponse à la question
La relation est : P = U . I.

■■Activité 2 Quelle relation permet de calculer l’énergie électrique ?


• Matériel
Un générateur.
Un joulemètre.
Une lampe adaptée.
Des cordons.

• Réponse aux questions


1. L’énergie reçue par la lampe est proportionnelle à la durée de fonctionnement.
2. Le produit P.t est voisin de l’énergie E lue sur le joulemètre.

Les exercices
Je sais 3. Unités pratiques
1. Q.C.M. Appareil Four Tondeuse Téléviseur
1 : A et B ; 2 : C ; 3 : A, B et C ; Énergie E
3 6 12
4 : A et B ; 5 : B et C ; 6 : B et C. (kWh)

2. Unités légales Puissance


2 3 0,5
P (kW)
Appareil Visseuse DEL rouge Calculatrice Durée t (h) 1,5 2 24
Énergie
E (J)
9 000 90 0,6 4. Puissance électrique

Puissance Lampe de Sèche- Moteur de


300 0,025 0,06 Appareil
P (W) scooter cheveux jouet
Durée Tension
30 3 600 10 12 230 4,5
t(s) (V)
Intensité
1,25 3,26 2
(A)
Puissance
15 750 9
(W)

CHAPITRE 5 - Puissance et énergie électriques • 17


5. Convertir P 2 200
b. On a P = U . I donc I =  = = 9,6 A.
1 500 W = 1,5 kW 21 kW = 21 000 W U 230
72 000 000 J = 20 kWh 0,25 kWh = 900 000 J 9. Réacteur nucléaire
6 min = 0,1 h 1 min30 s = 90 s Ce réacteur nucléaire peut alimenter :
1 h 15 min = 4 500 s 1 h 15 min = 1,25 h 109
= 2 500 000 ordinateurs.
400

J’applique 10. Lire une facture EDF


a. La facture est établie sur 6 mois.
6. Compteur d’énergie b. Consommation : 690 kWh.
a. E = 72 447 – 72 432 = 15 kWh.
c. Prix du kWh HT correspondant à la consomma-
b. E = P.t = 2,5 × 6 = 15 kWh.
tion : 54, 89 = 0,08 E.
7. Joulemètre 690
a. Le schéma du montage : d. Le prix TTC est : 94,52 + 11,57 = 106,09 E
Le prix de revient moyen du kWh TTC :
106, 09 = 0,15 E.
690
Joulemètre
11. Installation domestique
000
U t I a. Puissance totale disponible :
We
mode P W départ 1 500 + 2 500 = 4 000 W.
I A
arrêt
P 4000
U V b. Intensité efficace : I =  = = 17,4 A.
U 230
K 12. La rame V150
G
a. Puissance de la rame V150 :
P = 736 × 26 630 = 19 600 000 W soit 19 600 kW.
b. On a : E = P . t = 12 × 200 = 2 400 J.
b. Énergie consommée :
Le joulemètre indique : 2,40 kJ. 1
E = P.t = 19 600 × = 1 630 kWh.
8. Plaque signalétique 12
a. Les indications concernent les grandeurs électri- 13. Quel coût ?
ques nominales : a. Aspirateur : 1,2 × 0,5 × 0,15 = 0,09 E.
– la puissance 2 200 W ; b. Téléviseur : 0,6 × 3,25 × 0,15 = 0,29 E.
– la tension efficace 230 V ; c. Console de jeu : 0,2 × 0,75 × 0,15 = 0,02 E.
– la fréquence de la tension alternative : 50 Hz. d. Radiateur : 2 × 10 × 90 × 0,15 = 270 E.

LE DOCUMENT

■■Les lampes basse consommation (LBC)


• Réponse aux questions
1. Les lampes à basse consommation consomment moins d’énergie que celles à incandescence,
car elles chauffent moins.
2. Une lampe fluocompacte de 15 W est équivalente à une lampe à incandescence de 75 W.
3. On récupère les lampes fluocompactes hors d’usage, car elles contiennent du mercure : ce métal
est dangereux pour les différents organismes vivants.
4. Les lampes à DEL consomment moins d’énergie que les deux autres types de lampes et présentent
une durée de vie plus grande.

18 • CHAPITRE 5 - Puissance et énergie électriques


PRÉPARER LE CCF

Puissance d’une installation


Le tableau ci-dessous propose un barème d’évaluation pour aider à la notation de chaque élève.
Des fiches analogues, directement exploitables par l’enseignant, peuvent être téléchargées
sur le site de Hachette Éducation.

Vérifications Exploitation des résultats expérimentaux

1. Vérification du montage interrupteur Écriture de la relation entre P, P1 et P2 ....................... 1 pt


ouvert (branchement de l’ampèremètre, Traduction de la relation par une phrase ................. 1 pt
du voltmètre). ........................................................ 3 pts
Réponse aux questions
– La puissance distribuée par le générateur
2. Manipulation des interrupteurs
ou le secteur est égal à la somme
K1 et K2 . ..................................................................... 2 pts
des puissances consommées
par les appareils de l’installation ................................ 1 pt
3. Remise en état du poste de travail
– Pour le montage, on peut utiliser la loi
(sélecteurs sur zéro) ............................................ 1 pt
d’additivité pour les intensités
des courants (I = I1 + I2) ..................................................... 1 pt

TOTAL ....................................................................... 6 pts TOTAL .............................................................................. 4 pts

CHAPITRE 5 - Puissance et énergie électriques • 19


6 Prévention des risques électriques
Les activités
■■Activité 1 Protection des installations
• L’expérience
Lecture active (en relevant et explicitant les termes importants) du document.

• Réponse aux questions


1. Il s’agit du calibre des fusibles. Le calibre correspond à l’intensité maximale que peut supporter
le fusible.
2. Si le constructeur indique qu’il faut placer un fusible de 10 ampères, c’est parce que l’installation
est prévue pour supporter cette intensité maximale. Si l’intensité dépasse cette valeur, le fusible
coupe le circuit et protège l’installation. En plaçant un fusible de 15 A, le circuit sera coupé à partir de
15 ampères. L’installation risque donc d’être détériorée.
3. La fusion du fusible indique une surintensité. Dans cet exemple, la surintensité est due à un court-
circuit.

■■Activité 2 Protection des personnes


• L’expérience
Lecture active (en relevant et explicitant les termes importants) de l’énoncé et des légendes des
schémas.
Observation des deux documents.

• Réponses aux questions


Installation 1
1. Joé s’électrocute car, malgré la mise à la terre de la machine, le circuit reste fermé donc la machine
reste sous tension. Le courant traverse donc Joé pour aller à la terre. En quelque sorte, Joé sert de
deuxième « fil de terre ».
2. Non, la prise de terre seule ne suffit pas pour protéger les personnes.
Installation 2
3. Au moment du déclenchement du disjoncteur, le courant dans le fil de phase est différent de celui
dans le fil de neutre, car le fil de phase est en contact avec la carcasse de la machine. Une partie du
courant arrivant par le fil de phase repart donc par le fil de terre.
4. Joé ne peut pas s’électrocuter, car le disjoncteur ouvre le circuit dès l’apparition du défaut.
Une prise de terre associée à un disjoncteur différentiel protège les personnes.

20 • CHAPITRE 6 - Prévention des risques électriques


Les exercices
Je sais J’applique
1. Q.C.M. 6. Contacts directs et indirects
1- A, C ; 2- A ; 3- B, C ; 4- B ; 5- B, C ; 6- B ; 7- B. a.
2. Repérer les appareils de protection
a. La protection des personnes est assurée par  le
fil de terre et les deux disjoncteurs différentiels de
30 mA.
b. La protection des matériels est assurée par le para-
foudre et les quatre disjoncteurs divisionnaires.

3. Lire un schéma
a. Les appareils de protection des personnes sont : b. Ces situations présentent un danger d’électrocu-
le fil de terre ou conducteur de protection électrique tion.
(PE) schématisé par un trait vert et jaune et le disjonc- c. Il y a contact direct dans les deux premiers cas et
teur différentiel. contact indirect dans le troisième.
b. Les appareils de protection des matériels sont  : 7. Corriger le schéma
deux fusibles de calibre 10 ampères et 16 ampères. a. Le branchement du four convient mais il manque
4. Le bon calibre le conducteur de protection dans le branchement du
Les deux fusibles sont différents par leur calibre (10 A lave-linge.
ou 20 A). b.
a. Pour un four consommant 18 A, il faut un fusible disjoncteur
différentiel
de calibre 20 A.
b. Pour dix lampes en parallèle consommant cha-
cune 0,5 A, un fusible de calibre 10 A suffit. 32 A 20 A
10 A
c. L’intensité totale est égale à 12,3 A donc il faut utili-
ser une ligne protégée par un fusible de calibre 20 A.

5. Choisir la bonne réponse


Les bonnes réponses sont :
a. les fusibles sont branchés en série avec les appa- 8. Risques électriques
reils électriques ; a. 0,2 mA durant 2 secondes : aucun risque.
b. le contact avec le fil de phase est très dangereux ; b. 0,2 A (ou 200 mA) durant 2 secondes : zone rouge
c. le corps humain conduit le courant électrique ; donc fibrillation ventriculaire, arrêt cardiaque, graves
d. un disjoncteur divisionnaire (ou un fusible) de brûlures.
32 A protège parfaitement une ligne alimentant une c. Le disjoncteur différentiel de 100 mA doit déclen-
cuisinière qui absorbe 25 A. cher en moins de 700 ms ou 0,7 s pour éviter le risque
de fibrillation ventriculaire.

LE DOCUMENT

■■Choix des disjoncteurs divisionnaires dans une installation domestique


• Réponses aux questions
1. Pour protéger un chauffe-eau ou un four, il faut installer un disjoncteur de calibre 20 A.
2. Oui : le nombre de prises est limité à 5 prises sur un circuit dont le fusible a un calibre de 16 A et à
8 prises si le fusible a un calibre 20 A.
3. La section du câble alimentant une plaque de cuisson est de 6 mm².
4. L’intensité maximale consommée par une VMC est de 2 ampères.

CHAPITRE 6 - Prévention des risques électriques • 21


PRÉPARER LE CCF
Le coupe-circuit
Le tableau ci-dessous propose un barème d’évaluation pour aider à la notation de chaque élève.
Des fiches analogues, directement exploitables par l’enseignant, peuvent être téléchargées
sur le site de Hachette Éducation.

Vérifications Exploitation des résultats expérimentaux

1. Vérification du montage ............................... 1,5 pt Valeur de déclenchement................................................... 1 pt


Alimentation non branchée......................... 0,5 pt
Calibre 10 A ........................................................... 0,5 pt Calibre du fusible .................................................................... 1 pt
2. Positionnement du fusible
dans le circuit principal ................................ 0,5 pt Rôle du coupe-circuit ............................................................ 1 pt
Schématisation du circuit ................................ 1 pt
3. Vérification des résultats ............................... 2 pts

4. Remise en état du poste de travail ............. 1 pt

TOTAL ....................................................................... 7 pts TOTAL .............................................................................. 3 pts

22 • CHAPITRE 6 - Prévention des risques électriques


7 Prévention des risques chimiques
Les activités
■■Activité 1 Pourquoi doit-on lire l’étiquette d’un produit chimique ?
• L’expérience
Lecture active (en relevant et explicitant les termes importants pour répondre aux questions) de l’étiquette.
• Réponse aux questions
1. Phrase indiquant les risques : Irritant pour les yeux et la peau.
2. Le pictogramme dessiné signifie que le produit est irritant.
3. Les précautions à prendre sont :
- ne pas utiliser ce produit en combinaison avec d’autres produits ;
- conserver hors de portée des enfants ;
- ne pas respirer les vapeurs ;
- éviter le contact avec les yeux.

■■Activité 2 Comment manipuler en toute sécurité ?


• L’expérience
Observer chaque vignette et la mettre en corrélation avec les consignes notées sur la gauche de la page.

• Réponses aux questions 


Vignette 1 : Ne jamais diriger l’ouverture d’un tube à essai vers soi ou vers son voisin.
Vignette 2 : Ne jamais goûter ou sentir un produit.
Vignette 3 : Garder la paillasse propre et bien rangée.
Vignette 4 : Ne jamais pipeter à la bouche : utiliser un pipeteur. Garder la paillasse propre et bien rangée.

Les exercices
Je sais 3. Lire une étiquette
- Le pictogramme indique que le produit est corrosif.
1. QCM - La phrase de risque est : Provoque de graves brû-
1. B ; 2. A ; 3. C ; 4. A, B ; 5. B ; 6. B, C ; 7. A, B.
lures.
2. Informations légales - Le produit dangereux est de l’acide chlorhydrique.
a. Sur l’étiquette d’un produit, on doit trouver : - Avec ce produit il faut : le conserver hors de portée
- des phrases de risque, des enfants ; éviter le contact avec la peau et les yeux ;
- des phrases de sécurité, après contact avec la peau, se laver abondamment
- des pictogrammes. avec de l’eau et du savon.
b. Exemple de phrase de risque : - Il faut montrer l’étiquette au médecin en cas d’in-
Irritant pour la peau et les yeux. gestion pour qu’il prenne connaissance des produits
Exemple de phrase de sécurité : ingérés et des risques encourus.
Conserver sous clé, hors de portée des enfants.
4. Se protéger et protéger autrui
Pictogrammes :
a. Pour se protéger lors de la manipulation, Arthur
doit porter des gants, des lunettes et une blouse en
coton.
Inflammable Corrosif Comburant Explosif b. Il faut conserver le produit sous clé, hors de portée
des enfants et, si possible, dans un local correctement
ventilé.

Nocif Irritant Toxique Polluant

CHAPITRE 7 - Prévention des risques chimiques • 23


5. Choisir la bonne protection. mais aussi la conduite à tenir pour une utilisation du
a. Gérald peut utiliser une blouse en coton s’il ne produit concerné en toute sécurité.
souhaite pas se salir. Les autres dispositifs de protec- b. Damien doit porter des gants et des lunettes.
tion sont superflus.
b. Du fait de la présence de l’indicateur coloré, 7. Codes des phrases
Gérald doit vider la solution usagée dans un bidon de a. Le code R50 n’est pas noté, donc ce produit n’est
récupération. pas toxique pour les organismes aquatiques.
b. Ce produit ne doit pas être utilisé à proximité
J’applique d’une source de chaleur.
Il faut se protéger les yeux avec des lunettes de pro-
6. Attention aux flammes tection.
Premier test c. Il faut le conserver sous clé, hors de la portée des
a. Damien doit se placer sous la hotte aspirante allu- enfants et à l’écart de la chaleur. Il ne faut donc pas
mée. conserver ce produit sur une étagère du laboratoire.
b. Lors de l’utilisation de la flamme, Damien ne doit
pas utiliser de gants en latex et se placer à distance de 8. Bande dessinée
tout produit inflammable. a. Il se nettoie les mains avec un produit chimique
Second test dangereux.
a. Ce sont des pictogrammes qui informent sur les b. Il aurait dû utiliser un produit dégraissant, non
risques encourus lors de l’utilisation de ce produit dangereux pour sa santé.

LE DOCUMENT
■■Comment utiliser du déboucheur Destop sans risque ?
• Réponses aux questions
1. Il faut lire la notice et particulièrement les phrases de risque et de sécurité.
2. Le pictogramme indique le risque de brûlure, car le Destop est un produit corrosif.
3. Oui, car la phrase de risque notée est : «Provoque de graves brûlures. »
4. Il faut porter des gants, une blouse et des lunettes de protection.

PRÉPARER LE CCF
Diluer une solution
Le tableau ci-dessous propose un barème d’évaluation pour aider à la notation de chaque élève.
Des fiches analogues, directement exploitables par l’enseignant, peuvent être téléchargées
sur le site de Hachette Éducation.

Vérifications Exploitation des résultats expérimentaux

1. Risques encourus................................................... 1 pt Précautions notées ................................................................ 2 pts


Protections à porter.............................................. 1 pt
2. Choix du matériel de prélèvement ............. 1 pt
Utilisation du matériel ....................................... 1 pt
Précision du prélèvement ................................ 1 pt
3. Utilisation du matériel....................................... 1 pt
Précision..................................................................... 1 pt
4. Remise en état du poste de travail.............. 1 pt

TOTAL ....................................................................... 8 pts TOTAL .............................................................................. 2 pts

24 • CHAPITRE 7 - Prévention des risques chimiques


8 Élément chimique et atome
Les activités
■■Activité 1 Comment passer du cuivre métallique
au cuivre en solution ?
• Matériel
Un tube à essai dans un porte tube.
Une pipette.
Une hotte aspirante.
Une blouse, des gants et des lunettes de protection.

• Produits
Des copeaux de cuivre.
Une solution d’acide nitrique.

• Réponse aux questions


1. Le cuivre en copeaux est métallique.
2. La solution prend une couleur bleue.
3. Le cuivre métallique a disparu mais reste présent sous une autre forme dans la solution.

■■Activité 2 Quels sont les constituants de l’atome ?


• Réponse aux questions
1. Les deux parties de l’atome sont le noyau et le nuage d’électrons.
2. Le noyau contient des protons et des neutrons.
3. Le noyau a une charge positive, celle des protons.
4. Un électron a une charge négative.
5. La charge de l’ensemble des électrons est opposée à la charge du noyau : l’atome est donc
électriquement neutre.

Les exercices
Je sais 3. Charges électriques dans l’atome
1. Q.C.M. Atome Électron Noyau Proton Neutron
1 : A ; 2 : B ; 3 : B et C ; 4 : B ; 5 : A et C ; 6 : B ; 7 : C. électri- chargé chargé chargé électri-
quement négative- positive- positive- quement
2. Représentation de l’atome neutre ment ment ment neutre

4. Symbole des éléments


noyau proton
Nom de l’élément Symbole chimique
Oxygène O
électron Hydrogène H
Azote N
nuage neutron Potassium K
d’électrons Calcium Ca

CHAPITRE 8 - Élément chimique et atome • 25


5. Constituants de l’atome 10. Engrais NPK

Symbole de l’atome
12
C
1
H
16
O Nombre
6 1 8 Symbole Nom de Nombre Nombre
de
de l’atome l’élément de protons d’électrons
Nom de l’atome carbone hydrogène oxygène neutrons
14
Nombre de protons 6 1 8 7
N azote 7 7 7

31
Nombre d’électrons 6 1 8 15 P phosphore 15 15 16

Nombre de neutrons 6 0 8 40
19 K potassium 19 19 21

J’applique 11. Soudure à l’étain


a.
6. Symbole des atomes
Magnésium : Mg ; Nom de l’élément Cuivre Étain
sodium : Na ;
potassium : K ; Symbole chimique Cu Sn
zinc : Zn. Nombre de protons 29 50
7. Devinettes Nombre d’électrons 29 50
a. Le lithium.
b. L’aluminium. Nombre de neutrons 34 69
c. Le magnésium.
b. Le pictogramme signifie que le décapant utilisé
8. Électroneutralité des atomes par le plombier est un produit corrosif.
a. Un atome a une charge électrique nulle.
b.
12. Lampe fluocompacte
a.
Nom de
Oxygène Fluor Silicium Chlore Nom de l’élément Argon Néon
l’atome
Symbole Symbole chimique Ar Ne
de O F Si Cl
l’atome Symbole de l’atome 40
18P
20
10 P
Charge
8+ 9+ 14 + 17 + Nombre de protons 18 10
du noyau
Charge Nombre d’électrons 18 10
des 8– 9– 14 – 17 –
électrons b. Les pictogrammes représentant les dangers du
Charge mercure sont :
de nulle nulle nulle nulle D F A
l’atome

9. Symbole de l’atome de fer


a. Le nombre 56 représente la somme du nombre de
protons et de neutrons contenus dans le noyau d’un
atome de fer. Toxique Corrosif Dangereux
Le nombre 26 représente le nombre de protons dans pour
le noyau égal au nombre d’électrons autour du noyau l’environnement
de l’atome de fer. c. Les lampes fluocompactes usagées ne doivent pas
b. Le noyau de l’atome de fer contient 26 protons. être jetées aux ordures ménagères. Il faut les ramener
c. L’atome de fer a 26 électrons en mouvement aux points de vente pour qu’elles puissent être recy-
autour du noyau. clées.

26 • CHAPITRE 8 - Élément chimique et atome


LE DOCUMENT

■■Du minerai de fer à l’acier du cadre du BMX


• Réponse aux questions
1. Dans la nature, le fer se présente sous forme de minerai de fer.
2. Fe est le symbole chimique du fer.
3. Un atome de fer est composé de 26 protons, de 26 électrons et de 30 neutrons.
4. Ductile : qui peut s’étirer, s’allonger sans se rompre.
Malléable : que l’on peut façonner et réduire facilement en feuilles.
5. À 1 400 °C, le fer est solide.
6. Un acier contient moins de 2 % de carbone.
7. Un alliage est un métal composé d’un ou plusieurs éléments métalliques ou non.
8. Un acier trempé a le défaut d’être cassant.

PRÉPARER LE CCF
Les métaux
Le tableau ci-dessous propose un barème d’évaluation pour aider à la notation de chaque élève.
Des fiches analogues, directement exploitables par l’enseignant, peuvent être téléchargées
sur le site de Hachette Éducation.

Vérifications Exploitation des résultats expérimentaux

1. Vérification de la reconnaissance Nom du métal attiré par l’aimant ................................. 1 pt


des métaux par leurs couleurs..................... 2 pts
Nom du métal le plus léger
2. Vérification du test avec l’aimant ............. 2 pts (le moins dense) ................................................................... 0,5 pt

3. Vérification de la pesée.................................... 2 pts Nom de l’autre métal (le plus dense) ....................... 0,5 pt
Remise en état du poste de travail ............. 1 pt
Tableau des formules chimiques des éléments
correspondant à chaque métal ........................................ 1 pt

TOTAL ....................................................................... 7 pts TOTAL .............................................................................. 3 pts

CHAPITRE 8 - Élément chimique et atome • 27


9 Les molécules
Les activités
■■Activité 1 Quels sont les constituants d’une molécule ?
• Matériel
Une boîte de modèles moléculaires.

• Réponse aux questions


1. La molécule d’eau contient 2 atomes d’hydrogène et 1 atome d’oxygène. La molécule de dioxyde
de carbone contient 2 atomes d’oxygène et 1 atome de carbone.
2. H2O : l’indice 2 indique le nombre d’atomes d’hydrogène dans la molécule d’eau.
3. CO2 : l’indice 2 indique le nombre d’atomes d’oxygène dans la molécule de dioxyde de carbone.

■■Activité 2 Comment s’établissent les liaisons entre les atomes ?


• Matériel
Un jeu de cartes représentant les modèles de Lewis de deux atomes d’hydrogène, d’un atome de
carbone et de deux atomes d’oxygène.
Expérience

Modèle
Molécules Atomes Assemblage de Lewis de
la molécule

2 atomes
Dihydrogène
d’hydrogène H H H H

2 atomes H H
d’hydrogène
Eau O O
1 atome
d’oxygène H H

1 atome
Dioxyde de de carbone
carbone O C O O C O
2 atomes
d’oxygène

• Réponse aux questions


1. Un atome d’hydrogène possède un électron sur sa couche externe. Un atome de carbone possède
quatre électrons sur sa couche externe. Un atome d’oxygène possède six électrons sur sa couche
externe.
2. Dans le modèle de Lewis de la molécule d’eau, on observe un doublet d’électrons sur chaque
atome d’hydrogène et quatre doublets d’électrons sur l’atome d’oxygène.
3. Dans le modèle de Lewis de la molécule de dioxyde de carbone, on observe quatre doublets
d’électrons sur chaque atome d’oxygène et quatre doublets d’électrons sur l’atome de carbone.

28 • CHAPITRE 9 - Les molécules


Les exercices

Je sais 8. Calculer une masse molaire moléculaire


a. M(CO2) = M(C) + 2 × M(O) = 12 + 2 × 16
1. Q.C.M. = 44 g/mol.
1 : B ; 2 : B et C ; 3 : B ; 4 : A ; 5 : A ; 6 : C ; 7 : B.
b. M(CH4) = M(C) + 4 × M(H) = 12 + 4 × 1 = 16 g/mol.
2. Formule d’une molécule 9. Modèles moléculaires éclatés
H2, CH4, CO et CO2 représentent des formules de
Molécule de dihydrogène de formule H2 :
molécules.
modèle moléculaire 2.
3. Formule et nom d’une molécule Molécule de dioxyde de carbone de formule CO2 :
modèle moléculaire 4.
Nom de la molécule Formule Molécule de méthane de formule CH4 :
modèle moléculaire 1.
Dihydrogène • • O2
Molécule d’ammoniac de formule NH3 :
modèle moléculaire 3.
Dioxyde de carbone • • H2O

Eau • • H2 J’applique
10. Reconnaître des formules
Dioxygène • • CO2
a. Une molécule de dihydrogène : H2.
b. Deux molécules de dihydrogène : 2 H2.
4. Composition d’une molécule
c. Deux atomes d’hydrogène : 2 H.
La molécule d’aspirine de formule C9H8O4 est com-
d. Une molécule de chlorure d’hydrogène : HCl.
posée de 9 atomes de carbone, de 8 atomes d’hydro-
e. Une molécule d’eau : H2O.
gène et de 4 atomes d’oxygène.
11. Construire de modèles moléculaires
5. Écrire la formule
a.
Formule de l’acide acétique : C2H4O2.
Molécule Formule Composition Modélisation
6. Modèle atomique
2 atomes
Modèle d’hydrogène
Atome Symbole atomique : Eau H2O
1 atome
boule de couleur d’oxygène

Hydrogène H blanche 2 atomes 2 boules


Dihydrogène H2
d’hydrogène blanches
Carbone C noire
2 atomes 2 boules
Dioxygène O2
Oxygène O rouge d’oxygène rouges
1 atome de 1 boule
Chlore Cl verte Chlorure chlore blanche
HCl
Azote N bleue d’hydrogène 1 atome 1 boule
d’hydrogène verte
1 atome de 1 boule
7. Masse molaire atomique
Dioxyde de carbone noire
CO2
carbone 2 atomes 2 boules
Atome Masse molaire atomique en g/mol
d’oxygène rouges
Hydrogène 1 1 atome de 1 boule
carbone noire
Méthane CH4
Carbone 12 4 atomes 4 boules
d’hydrogène blanches
Oxygène 16
1 atome 1 boule
Chlore 35,5 d’azote bleue
Ammoniac NH3
3 atomes 3 boules
Azote 14 d’hydrogène blanches

b. Les molécules n’ont pas la même forme.

CHAPITRE 9 - Les molécules • 29


12. Modèles de Lewis de quelques molécules
Les modèles de Lewis des molécules données dans le tableau de l’exercice 11 sont :

O
H H O O H Cl
H H
Eau Dihydrogène Dioxygène Chlorure d’hydrogène
H

H C H H N H
O C O H H
Dioxyde de carbone Méthane Ammoniac

13. Combustion du butane


a. La molécule de butane est composée de 4 atomes de carbone et de 10 atomes d’hydrogène.
b. La masse molaire moléculaire du butane est :
M(C4H10) = 4 × M(C) + 10 × M(H)
= 4 × 12 + 10 × 1 = 58 g/mol.

LE DOCUMENT

■■La chimie dans les aliments : les fermentations


• Réponse aux questions
1. La levure de boulanger permet la fermentation du sucre présent dans la farine.
2. L’expérience montre que la fermentation de la farine dans de l’eau avec de la levure produit du
dioxyde de carbone.
3. Les trous dans la mie de pain sont produits par le dioxyde de carbone piégé dans la mie de pain
lors de sa fermentation.
4. La fermentation alcoolique du jus de raisin est représentée par les schémas suivants.

ballon en plastique dioxyde de carbone


souple

raisins écrasés
elle se trouble

eau tiède eau de chaux

On place un ballon souple en plastique sur le goulot d’un flacon contenant des grains de raisins
écrasés. Au bout d’un ou deux jours, le ballon se gonfle (schéma 1). Le gaz contenu dans le ballon
trouble l’eau de chaux (schéma 2) : c’est du dioxyde de carbone.
5. Du dioxyde de carbone se dégage dans la fermentation de la pâte à pain et dans la fermentation
alcoolique du jus de raisin.

30 • CHAPITRE 9 - Les molécules


PRÉPARER LE CCF
Eau minérale gazeuse
Le tableau ci-dessous propose un barème d’évaluation pour aider à la notation de chaque élève.
Des fiches analogues, directement exploitables par l’enseignant, peuvent être téléchargées
sur le site de Hachette Éducation.

Vérifications Exploitation des résultats expérimentaux

1. Vérification du test de mise L’eau minérale contient de l’eau ................................... 1 pt


en évidence de l’eau.......................................... 3 pts
Aspect de l’eau de chaux et nom du gaz ................... 1 pt
2. Vérification du test de mise
en évidence du dioxyde de carbone ....... 3 pts
Nom et nombre des atomes constituant
Remise en état du poste de travail ............. 1 pt
les deux molécules . ................................................................. 1 pt

TOTAL ....................................................................... 7 pts TOTAL .............................................................................. 3 pts

CHAPITRE 9 - Les molécules • 31


10 Les solutions ioniques
Les activités
■■Activité 1 Quels sont les ions présents dans une eau minérale ?
• Matériel
Plusieurs étiquettes d’eaux minérales.

• Réponse aux questions


1. L’ion sulfate est constitué de 1 atome de soufre et de 4 atomes d’oxygène.
2. L’indice 4 indique le nombre d’atomes d’oxygène.
3. L’ion sulfate est un ion négatif.
4. L’ion sulfate porte 2 charges électriques négatives.

■■Activité 2 Comment identifier les ions en solution ?


• Matériel
Un porte tube avec des tubes à essai.

• Produits
Une solution de chlorure de sodium.
Une solution de sulfate de cuivre.
Une solution de sulfate de fer II.
Une solution de chlorure de fer III.
Une solution de nitrate d’argent dans un flacon compte-gouttes.
Une solution de soude dans un flacon compte-gouttes.

• Réponse aux questions


Expérience 1
1. Le nitrate d’argent permet d’identifier les ions chlorure.
2. Le précipité est blanc.
3. Le précipité noircit lorsque le tube reste exposé à la lumière.
Expérience 2
4. La soude permet de caractériser les ions métalliques :
- le précipité est bleu avec les ions cuivre ;
- le précipité est vert avec les ions fer II ;
- le précipité est rouille avec les ions fer III.

32 • CHAPITRE 10 - Les solutions ioniques


Les exercices

Je sais e. Ce gaz est du dioxyde de carbone, car il trouble


l’eau de chaux. Sa formule est CO2.
1. Q.C.M.
1 : B et C ; 2 : B ; 3 : B ; 4 : A ; 5 : C. 7. La bouillie bordelaise
a. Un fongicide est une substance qui détruit les
2. Écrire les formules des ions champignons microscopiques.
Ion fer III : Fe3+ ; ion chlorure : Cl- ;
b. Le précipité blanc obtenu avec le chlorure de
ion cuivre : Cu2+; ion fer II : Fe2+.
baryum permet d’identifier les ions sulfate de for-
2–
3. Caractériser des ions mule SO 4 , le précipité bleu avec la soude identifie
les ions cuivre de formule Cu2+.
Formule Nom Couleur c. Les ions cuivre Cu2+ donnent la couleur bleue à la
de l’ion du réactif du précipité
bouillie bordelaise.
Cl– nitrate d’argent blanc
8. Les produits anti-mousse
Cu2+ soude bleu 2–
a. La formule des ions sulfate est SO 4  ; celle des ions
Fe2+ soude vert fer II est Fe2+.
Fe3+ soude rouille b. Test des ions sulfate : verser 2 à 3 mL de sulfate de
fer dans un tube à essai et y ajouter quelques gout-
Ca2+ oxalate d’ammonium blanc
tes de chlorure de baryum ; on observe un précipité
2–
SO 4 chlorure de baryum blanc blanc caractéristique des ions sulfate en solution.
Test des ions fer II : verser 2 à 3 mL de sulfate de fer
4. Décrire l’ion nitrate dans un tube à essai et y ajouter quelques gouttes de
a. L’ion nitrate est constitué de 1 atome d’azote et de soude ; on observe un précipité vert caractéristique
3 atomes d’oxygène. des ions fer II en solution.
b. Sa charge est négative ; il porte 1 charge négative. Schémas des tests :
Solution de Soude
5. Identifier des ions chlorure de
a. Le produit insoluble qui se forme lors de la carac- baryum
térisation des ions est un précipité.
b. Tube a : le précipité est bleu, on a identifié les ions
cuivre de formule Cu2+. Précipité Précipité
blanc vert
Tube b : le précipité est rouille, on a identifié les ions
fer III de formule Fe3+. Solution de Solution de
Tube c  : le précipité est vert, on a identifié les ions sulfate de fer sulfate de fer
fer II de formule Fe2+. 9. Identifier une solution
a. L’ion sodium est identifié dans l’expérience 1, car
J’applique la flamme est jaune. Sa formule est Na+.
6. Composition d’une eau minérale b. Dans l’expérience 2, on a identifié l’ion chlorure,
a. L’ion bicarbonate comporte 1 atome d’hydrogène, car on obtient un précipité blanc avec le nitrate d’ar-
1 atome de carbone et 3 atomes d’oxygène. gent. Sa formule est Cl-.
C’est un anion car il porte une charge négative. c. C’est une solution de chlorure de sodium.
2–
b. Ions chlorure : Cl- ; ions sulfate : SO 4  ; ions fluo- 10. De l’atome à l’ion
rure : F- ;
ions sodium : Na+ ;
ions potassium : K+ ; ions a. Tableau :
calcium : Ca ; ions magnésium : Mg2+.
2+ 
Ca Ca2+
c. Les ions calcium sont identifiés avec une solution
d’oxalate d’ammonium ; les ions chlorure avec une Nombre de protons 20 20
solution de nitrate d’argent et les ions sulfate avec Nombre d’électrons 20 18
une solution de chlorure de baryum.
Avec ces trois réactifs, on observe des précipités b. En perdant 2 électrons, l’atome de calcium perd
blancs. 2 charges négatives et devient un ion positif avec un
d. Cette eau contient un gaz dissous, car l’étiquette excès de 2 charges positives.
indique quelle est regazéifiée avec son propre gaz. c. Ion sodium : Na+. d. Ion chlorure : Cl-.

CHAPITRE 10 - Les solutions ioniques • 33


LE DOCUMENT

■■Les sels minéraux dans les aliments


• Réponse aux questions
1. L’organisme assimile les sels minéraux dont il a besoin sous forme d’ions. Il les trouve dans les
aliments.
2. a. L’ion phosphate est constitué de 1 atome de phosphore et de 4 atomes d’oxygène.
b. L’ion phosphate porte 3 charges négatives.
3. Un oligo-élément est un élément chimique nécessaire, en infime quantité, à la vie des êtres
humains et des végétaux.
4. a. Un enfant ayant une fragilité osseuse présente une carence en calcium. L’apport quotidien
recommandé est de 800 mg.
b. Les produits laitiers apportent la quantité nécessaire de calcium.

PRÉPARER LE CCF

Dureté de l’eau
Le tableau ci-dessous propose un barème d’évaluation pour aider à la notation de chaque élève.
Des fiches analogues, directement exploitables par l’enseignant, peuvent être téléchargées
sur le site de Hachette Éducation.

Vérifications Exploitation des résultats expérimentaux

1. Vérification du test des ions Classement des trois eaux minérales,


calcium dans les trois tubes.......................... 3 pts de la plus douce à la plus dure,
à partir du test des ions calcium .................................... 1 pt
2. Vérification de l’utilisation
Classement des trois eaux minérales,
des bandelettes AQUADUR . ........................ 3 pts
de la plus douce à la plus dire,
3. Remise en état du poste de travail ............. 1 pt à partir des bandelettes........................................................ 1 pt
Nom de l’eau minérale la plus douce
et nom de l’eau minérale la plus dure ..................... 0,5 pt
Vérification des classements ......................................... 0,5 pt

TOTAL ....................................................................... 7 pts TOTAL .............................................................................. 3 pts

34 • CHAPITRE 10 - Les solutions ioniques


11 Solutions et concentrations
Les activités
■■Activité 1 Comment calculer la concentration massique d’une solution ?
• Matériel
Une balance électronique à 0,1 g.
Une capsule.
Une spatule.
Une fiole jaugée de 50 mL munie d’un bouchon.
Un entonnoir.

• Produits
De la tartrazine en poudre.
De l’eau distillée.

• Réponse aux questions


1. 0,2 × 20 = 4 g. Un litre de solution contiendrait 4 g de tartrazine.
2. La concentration massique de la tartrazine dans la solution est de 4 g/L.

■■Activité 2 Comment calculer la concentration molaire d’une solution ?


Expérience

Nombre de moles n dans 1 litre


Masse de soluté m
de solution
dans 1 litre de
solution n= m
534, 4
Solution (a) m = 4 g 0,007 mol

Solution (b) m=2g 0,004 mol

Solution (c) m=1g 0,002 mol

• Réponse aux questions


1. Le nombre de moles dissous dans un litre de solution est :
- solution (a) : 0,007 mol de tartrazine ;
- solution (b) : 0,004 mol de tartrazine ;
- solution (c) : 0,002 mol de tartrazine.
2. La concentration molaire des trois solutions est :
- solution (a) : 0,007 mol/L de tartrazine ;
- solution (b) : 0,004 mol/L de tartrazine ;
- solution (c) : 0,002 mol/L de tartrazine.
3. La solution (a), dont la concentration est la plus grande, est jaune foncé ;
la solution (c) de concentration la plus faible est jaune clair.
4. La concentration molaire de la solution de tartrazine diminue lorsque le volume d’eau augmente.

CHAPITRE 11 - Solutions et concentrations • 35


Les exercices

Je sais c. n s’exprime en mole (symbole : mol) ; V s’exprime


en litre (symbole : L) ; C s’exprime en mole par litre
1. Q.C.M. (symbole : mol/L).
1 : A et C ; 2 : A ; 3 : C ; 4 : C ; 5 : B ; 6 : A et C.

2. Grandeurs et unités J’applique


Grandeurs Unités 4. Solutions de sulfate de cuivre
Masse • • g/L a. 25 mL = 0,025 L ; 50 mL = 0,05 L ; 100 mL = 0,1 L.
b. Concentration massique en sulfate de cuivre :
Volume • •L m 5
- solution 1. : Cm =  = = 200 g/L  ;
Masse molaire • • mol/L V 0,025
- solution 2. : 100 g/L ;
Concentration massique • •g - solution 3. : 50 g/L.
c. La solution la plus foncée est la solution 1, car c’est
Concentration molaire • • g/mol
la solution la plus concentrée en sulfate de cuivre.
3. Exprimer une formule 5. Lire une étiquette d’eau minérale
a. Il s’agit d’une concentration molaire. a. Les valeurs exprimées en mg/L indiquent des
b. La lettre n représente le nombre de moles de concentrations massiques.
soluté. b. Concentrations massiques des ions présents dans
La lettre V représente le volume de la solution. l’eau minérale :

Ions Calcium Magnésium Sodium Potassium Bicarbonates Sulfate Chlorure Fluorure

Concentrations
massiques
0,09 g/L 0,011 g/L 1,708 g/L 0,132 g/L 4,368 g/L 0,174 g/L 0,322 g/L 0,001 g/L
en g/L

Les deux sels minéraux majoritaires dans cette eau 8. Produit ménager
minérale sont les bicarbonates et le sodium. a. Liste du matériel  nécessaire  : une balance élec-
c. 90 × 0,15 = 13,5. tronique au centième de gramme, une spatule, une
Lorsqu’on boit un verre de 15 cL de cette eau miné- capsule, une fiole jaugée de 1 L munie d’un bouchon,
rale, on absorbe 13,5 mg de calcium. un entonnoir, une pissette d’eau distillée.
b. m = 56 × 0,01 = 0,56. La masse de 0,01 mol d’hy-
6. Dureté d’une eau minérale droxyde de potassium est égale à 0,56 g.
C(Ca 2+ ) C(Mg 2+ ) 203,8 43,1 c. Pour réaliser un litre de la solution, il faut :
a. D =  + =  + ≈ 69.
4 2,4 4 2,4 - prélever avec une spatule et peser 0,56 g d’hydroxyde
La dureté de l’eau de Vittel est égale à 69 °HT. de potassium dans une capsule ;
b. L’eau de Vittel est qualifiée de dure car sa dureté - introduire les 0,56 g d’hydroxyde de potassium dans
est supérieure à 15 °HT. la fiole jaugée de 1 L à l’aide d’un entonnoir ;
- ajouter de l’eau distillée en restant au-dessous du
7. Sirop de grenadine trait de jauge ;
m 77 - boucher la fiole jaugée et agiter doucement pour
a. Cm =  = = 770. homogénéiser la solution ;
V 0,1
- compléter avec de l’eau distillée jusqu’au trait de
La concentration massique de sucre dans le sirop de
jauge.
grenadine est de 770 g/L.
b. Le sirop a été dilué 8 fois. Sa concentration dans le 9. Décaper du carrelage
verre n’est plus que 770/8 = 96 mg/L a. Le produit est de l’acide chlorhydrique (ou chlo-
c. 96 × 0,02 = 1,9. rure d’hydrogène) de formule HCl.
On absorbe 1,9 mg de sucre lorsqu’on boit un verre b. Le produit provoque des brûlures et il est irritant
de 20 mL du sirop dilué. pour les voies respiratoires.

36 • CHAPITRE 11 - Solutions et concentrations


c. Pour manipuler le produit, il faut porter une blouse Volume total : V = 200 + 800 = 1 000 mL = 1 L.
en coton, des gants et des lunettes de protection. La concentration molaire de la solution diluée d’acide
d. n = 0,2 × 0,2 = 0,04 mol. chlorhydrique obtenue est donc :
200  mL du produit contiennent 0,04 mol d’acide C = 0,04 mol/L.
chlorhydrique.

LE DOCUMENT

■■Analyse de l’eau de boisson


• Réponse aux questions
1. Les eaux diffèrent par leurs teneurs en minéraux qui sont différentes, leurs concentrations de
substances (pesticides, chlore, plomb …), leur pH, leur dureté.
2. Le résidu sec de l’eau d’Évian est de 309 mg/L. C’est une eau peu minéralisée.
3. Le savon mousse moins avec une eau dure et cela entraîne une surconsommation de lessive,
source de pollution supplémentaire. Une eau dure entartre les canalisations et les tuyauteries.
4. L’eau d’Évian est dure car sa dureté est supérieure à 15 °HT.
L’eau d’Évian respecte les critères de potabilité.

PRÉPARER LE CCF

Dilution d’une solution


Le tableau ci-dessous propose un barème d’évaluation pour aider à la notation de chaque élève.
Des fiches analogues, directement exploitables par l’enseignant, peuvent être téléchargées
sur le site de Hachette Éducation.

Vérifications Exploitation des résultats expérimentaux

1. Vérification du volume prélevé Précautions à prendre avant de manipuler . ....... 0,5 pt


avec la pipette jaugée.................................... 2,5 pts Nom du matériel nécessaire à la préparation
2. Vérification du remplissage d’une solution par dilution ............................................... 1 pt
de la fiole jaugée . ............................................ 2,5 pts Reconnaissance de la concentration ....................... 0,5 pt
Vérification du stockage Comparaison de la couleur de la solution

de la solution dans un flacon de permanganate de potassium préparée
soigneusement étiqueté.................................... 1 pt à celle de l’eau de Dakin . ................................................. 0,5 pt
Remise en état du poste de travail ............. 1 pt
Nom de l’espèce chimique responsable
de la couleur de la solution de Dakin . ..................... 0,5 pt

TOTAL ....................................................................... 7 pts TOTAL .............................................................................. 3 pts

CHAPITRE 11 - Solutions et concentrations • 37


12 Solution acide, neutre ou basique
Les activités
■■Activité 1 Comment mesurer le pH d’une solution ?
• Matériel pour un groupe d’élèves
Une capsule en verre.
Un agitateur en verre.
Trois béchers.
Un pH mètre.
Du papier pH avec le nuancier.

• Produits
Du vinaigre.
De l’eau pure.
De l’eau avec de la lessive.

• Réponse aux questions


1. Le classement des solutions par pH croissant : vinaigre ; eau pure ; eau de lessive.
2. Solution acide : le vinaigre ; solution neutre : l’eau pure ; solution basique : l’eau de lessive.

■■Activité 2 Comment varie le pH lors d’une dilution ?


• Matériel
Trois béchers de 100 mL.
Une éprouvette graduée de 100 mL.
Un agitateur en verre.
Un pH mètre.

• Produits
Une solution d’acide chlorhydrique à 0,1 mol/L.
De l’eau distillée.
Une solution d’hydroxyde de sodium à 0,1 mol/L.

• Réponse aux questions


Expérience 1
1. La solution A possède le pH le moins élevé, c’est la solution la plus acide.
2. Le pH d’une solution acide augmente lorsqu’on la dilue.
Expérience 2
3. La soude est une solution basique, car son pH est supérieur à 7.
4. La solution A possède le pH le plus élevé, c’est la solution la plus basique.
5. Le pH d’une solution basique diminue lorsqu’on la dilue.

38 • CHAPITRE 12 - Solution acide, neutre ou basique


Les exercices

Je sais 8. Reconnaître les symboles de danger


a. L’acide chlorhydrique est une solution irritante ;
1. Q.C.M. la lessive de soude est une solution corrosive.
1 : A et C 2 : C 3 : B et C
b. Pour manipuler ces deux solutions, il faut éviter
4 : A, B et C 5 : A et B 6 : B.
tout contact avec la peau et les yeux, ne pas l’inhaler
2. Produit Destop et ne pas l’avaler. Il faut porter des gants et des lunet-
Le Destop contenant de la soude caustique est une tes de protection ainsi qu’une blouse.
solution basique.
9. Effets de la dilution
3. Étudier une étiquette a. Le pH de la solution diluée d’acide chlorhydrique
a. La boisson Sprite contient de l’acide citrique. est supérieur à 3 et inférieur à 7.
b. On peut vérifier cette caractéristique en mesurant b. Le pH de la solution diluée de soude est inférieur à
son pH. 11 et supérieur à 7.

4. Shampooing 10. Analyser des mesures


L’expression « le produit a un pH neutre » signifie que a. Graphique ci-dessous :
le shampooing est une solution neutre, il n’est donc
pas corrosif pour le cuir chevelu. pH

5. Eau de Javel
a. La solution d’eau de Javel est basique.
b. Lorsque l’eau de Javel est diluée dans de l’eau, son
pH diminue.

J’applique le cours 7
6. Classer selon le pH
a. Solutions acides : jus de citron concentré ; vinai-
gre ; eau de pluie.
Solution neutre : eau de Volvic.
Solutions basiques : eau de mer ; sang ; lessive.
b. La solution la plus acide est le jus de citron  ; la
solution la plus basique est la lessive.
1
c. Teinte du papier pH avec chacune des solutions volume (mL)
• Jus de citron : jaune. 0
0 2 4 6 8 10
• Vinaigre : jaune.
• Eau de mer : mauve. b. Le pH de la solution se rapproche de 11,3.
• Sang : vert clair. c. La solution est basique.
• Lessive : violet.
• Eau de pluie : vert foncé. 11. Indicateurs colorés
• Eau de Volvic : vert clair. a. Saint Marc dégraissant : pH = 9 ;
Sun rinçage : pH = 2 ;
7. Batterie de voiture Produit nettoyant pour métaux : pH = 12.
a. Le pH de cette solution est plus petit que 7. b. Saint Marc dégraissant : basique ;
b. Le pH augmente. La solution devient moins Sun rinçage : acide ;
acide. Produit nettoyant pour métaux : basique.

CHAPITRE 12 - Solution acide, neutre ou basique • 39


LE DOCUMENT

■■L’eau et les boissons


• Réponse aux questions
1. L’acidité du jus de citron est due à la présence d’acide citrique.
2. Le pH mesure l’acidité d’une boisson.
3. On mesure l’acidité d’une boisson à l’aide de papier pH ou d’un pH mètre.
4. Le dioxyde de carbone est dissous dans les boissons gazeuses. Sa formule chimique est CO2.
Ce gaz rend ces solutions acides.
5. Sur l’échelle de pH, les boissons du tableau ci-dessous sont classées par ordre de pH croissant.

Oasis Vin

Champagne
1 2 3 4 5 6 7 pH

Coca-cola
Limonade Schweppes Eau de Perrier

Orangina Jus de citron

PRÉPARER LE CCF

Préparation d’une solution et mesure de son pH


Le tableau ci-dessous propose un barème d’évaluation pour aider à la notation de chaque élève.
Des fiches analogues, directement exploitables par l’enseignant, peuvent être téléchargées
sur le site de Hachette Éducation.

Vérifications Exploitation des résultats expérimentaux

1. Vérification de la sécurité.................................. 1 pt Réponses aux questions :

2. Vérification du remplissage – caractère de la solution .................................................... 1 pt


de la fiole jaugée : – évolution du pH lors de la dilution ........................... 1 pt
– prélèvement des 10 mL .............................. 1,5 pt – calcul de la concentration molaire
– ajustage au trait de jauge .......................... 1,5 pt de la solution 2 ........................................................................... 1 pt
Vérification de la mesure du pH
(utilisation du pH mètre et mesure)........ 2 pts

3. Remise en état du poste de travail ............. 1 pt

TOTAL ....................................................................... 7 pts TOTAL .............................................................................. 3 pts

40 • CHAPITRE 12 - Solution acide, neutre ou basique


13 Techniques d’analyse et de dosage
Les activités
■■Activité 1 Comment réaliser une chromatographie sur couche mince ?
• Matériel
Un bécher.
Une coupelle en verre.
Une feuille de papier à dessin.
Une pipette Pasteur.

• Produits
Eau salée.
Colorants E102 et E131.
Du sirop de menthe.

• Réponse aux questions


1. Les taches ne sont pas entraînées à la même hauteur. Le colorant jaune E102 migre le plus vite et le
plus haut.
2. Le sirop de menthe vert donne 2 taches distinctes.
3. La chromatographie permet de séparer et d’identifier certaines espèces contenues dans une
solution. Pour les reconnaître, il faut comparer les taches obtenues à celles données par des espèces
connues et qui servent de témoin.

■■Activité 2 Comment réaliser un dosage acide-base ?


• Matériel
Un support avec noix de serrage et pince.
Un agitateur magnétique.
Un barreau aimanté (turbulent).
Une burette.
Un bécher.

• Produits
Une solution de Harpic acide diluée 100 fois.
Une solution de soude à 0,05 mol/L.
Réactif : bleu de bromothymol.

• Réponse aux questions


1. Pour prélever un volume précis, on utilise une pipette jaugée avec un pipeteur ; pour verser la
solution titrante, une burette ; pour homogénéiser le liquide dans le bécher, un turbulent animé par
l’agitateur magnétique.
2. À l’équivalence, nous avons trouvé VB = 10,2 mL.
La solution d’acide chlorhydrique dans la solution analysée a pour concentration :
C B .V B 0,05 ×10,2 ×10 −3
CA = = = 0,0255 mol/L.
VA 20 ×10 −3
La concentration en acide chlorhydrique dans le flacon de Harpic est 100 fois plus élevée,
soit : 2,55 mol/L.
3. L’unité de concentration molaire est la mole par litre (mol/L).

CHAPITRE 13 - Techniques d’analyse et de dosage • 41


Les exercices

Je sais b. CB : concentration molaire de la base ; VB : volume


de base à l’équivalence.
1. Q.C.M. c. Les concentrations molaires s’expriment en mole
1 : B 2 : C 3 : A et C 4 : C 5 : A 6 : A.
par litre (mol/L) et les volumes en litre (L).
2. Recherche d’ions
Réactif
J’applique
Formule Phénomène
Ion caractérisant 8. Solution de Dakin
de l’ion observé
l’ion
a. Concentrations molaires de la solution dans les
Oxalate Précipité tubes (du plus foncé au plus clair) :
Calcium Ca2+
d’ammonium blanc b.
Chlorure de Précipité Tube 1 2 3 4 5
Sulfate SO 24 −
baryum blanc
C
Précipité 1 × 10-4 8 × 10-5 6 × 10-5 4 × 10-5 2 × 10-5
Chlorure Cl- Nitrate d’argent (mol/L)
blanc

Précipité c. Le tube X possède la même couleur que le tube 4,


Fer II Fe2+ Soude
vert d’où sa concentration molaire en ions MnO 4−  :
4 × 10-5 mol/L.
Cuivre Cu2+ Soude Précipité bleu
9. Colorants alimentaires :
3. Tests à la flamme a. Colorant rouge : Rf = 11/33 = 0,33.
a. La flamme est jaune, la solution contient donc des Colorant bleu : Rf = 29/33 = 0,88.
ions sodium. Colorant jaune : Rf = 21/33 = 0,64.
b. Une tige métallique est plongée dans une solu- b. Le colorant vert contient deux colorants : le bleu et
tion contenant des ions sodium puis présentée à la le jaune.
flamme. c. Le colorant le plus soluble dans l’eau (le solvant)
est celui qui migre le plus : le colorant bleu.
4. Produits de jardinage
a. La solution 1 contient des ions cuivre, la solution 2 10. Dosage de l’acidité du lait
des ions fer. a. Volume de lait versé : VA = 10 mL.
b. Ions cuivre : Cu2+ ; ions fer II : Fe2+. b. La solution titrante est une solution de soude
(hydroxyde de sodium).
5. Technique de séparation
c. Au début de l’expérience, le lait est blanc. Même
a. C’est une chromatographie.
après ajout de la phénolphtaléine, il reste blanc, car
b. Schéma :
celle-ci est incolore pour un pH inférieur à 8,2.
d. L’indicateur coloré est la phénolphtaléine : il sert
à visualiser le virage à l’équivalence  : le nombre de
moles d’acide est alors égal au nombre de moles de
front du
solvant base.
e. À l’équivalence, la solution est rose violacée.
papier dessin taches
(phase fixe) f. Calcul de :
C B .V B 0,05 × 4 ×10 −3
CA = = = 0,02 mol/L.
solvant VA 10 ×10 −3
(phase mobile)
La concentration molaire en acide du lait est de
6. Rapport frontal 0,02 mol/L.
a. Mesures : h = 17 mm ; H = 26 mm. g. Masse d’acide lactique dans un litre de lait :
h 17 m = 90 CA = 90 × 0,02 = 1,8 g, soit 18 °D.
b. Le rapport frontal est : R = = ≈ 0,65.
H 26
f

7. Dosage acide-base 11. Dosage du vinaigre


a. CA : concentration molaire de l’acide ; VA : volume a. Volume de soude à l’équivalence : VB = 13 mL.
d’acide à l’équivalence. b. Volume de vinaigre prélevé : VA = 10 mL.

42 • CHAPITRE 13 - Techniques d’analyse et de dosage


Concentration de la solution titrante : CB = 0,1 mol/L. La concentration molaire en acide acétique de la
solution analysée est : CA = 0,13 mol/L.
C B .V B 0,1×13 ×10 −3 d. La concentration molaire en acide acétique du
c. C A = = = 0,13 mol/L.
VA 10 ×10 −3 vinaigre est : 10 CA = 1,3 mol/L.

LE DOCUMENT

■■La verrerie
• Réponse aux questions
1. 1 → flacon ; 2 → bécher ; 3 → erlenmeyer ; 4 → burette ; 5 → pipette jaugée ; 6 → éprouvette.
2. Il faut revêtir une blouse et, selon les produits à manipuler, des lunettes et des gants.
3. Lors d’un dosage acide-base, on prélève la solution de concentration inconnue avec la pipette
jaugée. On verse dans la burette la solution titrante.

PRÉPARER LE CCF
Analyse d’un bonbon
Le tableau ci-dessous propose un barème d’évaluation pour aider à la notation de chaque élève.
Des fiches analogues, directement exploitables par l’enseignant, peuvent être téléchargées
sur le site de Hachette Éducation.

Vérifications Exploitation des résultats expérimentaux

1. Réalisation de la recherche Réponse aux questions :


du glucose................................................................ 2 pts – la recherche du glucose ............................................... 0,5 pt
– la vitesse d’entraînement ........................................... 0,5 pt
2. Vérification de la chromatographie
– la nature des colorants ...................................................... 1 pt
du bonbon :
– les constituants du bonbon............................................. 1 pt
– préparer la cuve .............................................. 1,5 pt
– déposer les gouttes ....................................... 1,5 pt

3. Vérification des résultats
de la chromatographie....................................... 1 pt
Remise en état du poste de travail ............. 1 pt

TOTAL ....................................................................... 7 pts TOTAL .............................................................................. 3 pts

CHAPITRE 13 - Techniques d’analyse et de dosage • 43


14 Le mouvement rectiligne
Les activités
■■Activité 1 Comment reconnaître un état de mouvement ou de repos ?
• Réponse aux questions
1. Le conducteur est en mouvement par rapport au sol.
2. Le conducteur est au repos (immobile) par rapport à l’autocar.

■■Activité 2 Comment décrire le mouvement d’un solide ?


• Matériel
Un récipient cylindrique ou une grande éprouvette contenant de l’huile.
Une bille en acier.
Un chronomètre.
Une webcam, un caméscope ou un appareil photo numérique (facultatif).

• Expérience
Résultats obtenus :
d (m) 0,20 0,40 0,60
t (s) 0,25 0,50 0,75
v (m/s) 0,8 0,8 0,8

• Réponse aux questions


1. La trajectoire de la bille est rectiligne : la bille, attirée par le centre de la Terre, est soumise
à son poids et se dirige verticalement, vers le bas.
2. La vitesse moyenne de la bille est constante : le mouvement de la bille est uniforme.

Les exercices

Je sais d. En 1 seconde : 90 000 ÷ 3 600 = 25 m.


On dit que la vitesse moyenne de l’automobile est :
1. Q.C.M. v = 25 m/s.
1 : B et C ; 2 : B ; 3 : A, B et C ; 4 : B ;
5 : B et C ; 6 : B ; 7 : B. 4. Calculer une vitesse moyenne
a. Vitesse moyenne du camion :
2. Reconnaître un état de mouvement
ou de repos d 455
v= = = 70 km/h.
a. Yann est en mouvement par rapport à Guillem t 6,5
assis sur le banc.
b. Durée du parcours de l’athlète :
b. Yann est au repos par rapport à Mathilde placée
14 min 25 s = 865 s.
dans le même bateau.
c. Le banc, le bateau sont des référentiels. D’où sa vitesse moyenne :

3. Conversion d’unités v=
d 5000
= = 5,78 m/s.
a. En 1 heure, elle parcourt 90 kilomètres. t 865
b. 90 km = 90 000 m. soit 5,78 × 3,6 = 20,8 km/h.
c. 1 h = 3 600 s.

44 • CHAPITRE 14 - Le mouvement rectiligne


5. Durée, vitesse, distance La vitesse est constante sur le parcours CF (12,5 m/s
a. d = v. t = 3,5 × 20 = 70. La distance parcourue est de ou 45 km/h) : le mouvement du skieur est bien uni-
70 m. forme.
b. 45 km/h = 12,5 m/s.
9. Vitesse sur autoroute
c. La durée du parcours :
a. Durée du parcours :
d 7500
t= = = 600 s ou 10 min. 10 h 50 min – 9 h 20 min = 1 h 30 min.
v 12,5
b. Distance totale parcourue :
6. Lecture d’une vitesse instantanée
44 613 – 44 406 = 207 km.
a. L’appareil est un compteur de vitesse ou cinémo-
c. Vitesses moyennes  sur les différentes parties du
mètre; il est gradué en kilomètre par heure (km/h).
parcours (on note 1, 2, 3 les positions du véhicule
b. La vitesse instantanée
correspondant aux lectures des cadrans) :
v = 65 km/h soit 65 ÷ 3,6 = 18,1 m/s.
44448 − 44406
v12 = = 126 km/h.
J’applique 1
3
7. Reconnaître la nature d’un mouvement
Les distances parcourues pendant des intervalles de 44613 − 44448
v 23 = ≈ 141,4 km/h.
temps égaux (distances entre les croix jaunes) dimi- 1
1+
nuent : le mouvement du scooter est ralenti. 6
207
8. Lecture d’un diagramme v13 = = 138 km/h.
1,5
Portion AC
a. Vitesses moyennes sur AB et BC : Contrairement à ce que les vitesses instantanées affi-
20 chées par le compteur le laissent supposer, la vitesse
v AB = = 4,55 m/s. varie tout au long du parcours.
4,4 − 0
20
v BC = = 10,53 m/s. 10. Construction de diagrammes
6,3 − 4,4
a. Les dates :
La vitesse augmente, le mouvement est accéléré. phase 1 : 0 à 2 secondes,
Portion CF phase 2 : 2 à 7 secondes ;
phase 3 : 7 à 10 secondes.
20 b. Phase 1 : la vitesse augmente donc le mouvement
b. vCD = 7,9 − 6,3 = 12,5 m/s.
est accéléré ; phase 2 : la vitesse est constante donc le
20 mouvement est uniforme ; phase 3 : la vitesse diminue
v DE = = 12,5 m/s.
9,5 − 7,9 donc le mouvement est ralenti et l’outil s’immobilise.
c. Distance parcourue par l’outil pendant la
20 phase 2 (phase de travail) :
v EF = = 12,5 m/s.
11,1 − 9,5 d = v. t = 5 × 5 = 25 cm.

LE DOCUMENT

■■Le chronotachygraphe dans les transports routiers


• Réponse aux questions
1. Les véhicules de plus de 3,5 tonnes de poids maximum autorisé (P.M.A.) et les véhicules
transportant plus de 9 personnes sont équipés de chronotachygraphes.
2. Lecture du disque :
Heure de départ : 9 h ;
Heure d’arrivée : 18 h 30 min ;
Distance totale parcourue : 144 227 – 143 812 = 415 km.
3. Vitesse maximale atteinte : 109 km/h.

CHAPITRE 14 - Le mouvement rectiligne • 45


PRÉPARER LE CCF

Chute d’une goutte de permanganate de potassium dans l’huile


Le tableau ci-dessous propose un barème d’évaluation pour aider à la notation de chaque élève.
Des fiches analogues, directement exploitables par l’enseignant, peuvent être téléchargées
sur le site de Hachette Éducation.

Vérifications Exploitation des résultats expérimentaux

1. Vérification de la mesure du temps Vitesse moyenne


au point A.................................................................... 1 pt La vitesse moyenne est constante . ............................... 1 pt
2. Vérification des calculs :
Vitesse moyenne durant les trajets Nature du mouvement
OA ................................................................................ 1,5 pt Le mouvement de la goutte de permanganate
OB ................................................................................ 1,5 pt de potassium est uniforme . ............................................. 2 pts
OC . .............................................................................. 1,5 pt
OD ............................................................................... 1,5 pt

TOTAL ....................................................................... 7 pts TOTAL .............................................................................. 3 pts

46 • CHAPITRE 14 - Le mouvement rectiligne


15 Le mouvement circulaire
Les activités
■■Activité 1 Comment mesurer une fréquence de rotation ?
• Matériel
Un moteur électrique muni d’un disque sur son axe.
Un tachymètre numérique à capteur mécanique et (ou) à capteur photoélectrique.

• Réponse aux questions


1. Le tachymètre est gradué en tour par minute (tr/min) ou en unité anglo-saxonne, en « rotation par
minute » (RPM).
2. Indiquer la valeur lue sur le cadran lors de l’expérience réalisée en classe.
Dans le document 1, la fréquence de rotation mesurée est n = 3 066 tr/min.

■■Activité 2 Quelle est la relation entre la vitesse moyenne


et la fréquence de rotation ?
• Matériel
Une roue de bicyclette sur sa fourche.
Un tachymètre numérique.
Un cyclomètre (compteur de vitesse) : capteur sur un rayon et boîtier visible.

• Expérience
Pour la roue : R = 33,5 cm, D = 67 cm, d’où une circonférence L = 210 cm.
Il faut vérifier que c’est bien la valeur programmée sur le cyclomètre.
Vitesse indiquée par le cyclomètre : 11 km/h soit v = 3,05 m/s.
Fréquence de rotation indiquée par le tachymètre : 89 tr/min soit n = 1,48 tr/s.
Calcul de l’expression :
π . D . n = π × 0,67 × 1,48 ≈ 3,12 m/s.

• Réponse à la question
La vitesse moyenne d’un point du pneu (3,05 m/s) est pratiquement égale de la valeur de
l’expression π . D . n (3,12 m/s).
On peut écrire : v = π . D . n.

Les exercices
Je sais Mouvement circulaire accéléré  : 3 car les distances
entre les points successifs (et les angles au centre
1. Q.C.M. balayés) augmentent avec le temps.
1 : B et C ; 2 : B et C ; 3 : A ; 4 : B ; 5 : A et C ;6 : A et C.
Mouvement circulaire ralenti  : 2 car les distances
2. Lecture d’une fréquence de rotation entre les points successifs (et les angles au centre
a. L’appareil dessiné est un tachymètre. balayés) diminuent avec le temps.
b. La lecture du cadran doit être multipliée par 100.
4. Utiliser une formule
c. Fréquence de rotation :
a. v  : vitesse moyenne exprimée en mètre par
n = 35 × 100 = 3 500 tr/min.
seconde ;
3. Nature du mouvement D : diamètre de la trajectoire circulaire, exprimé en
Mouvement circulaire uniforme : 1 car les distances mètre ;
entre les points successifs (et les angles au centre n  : fréquence de rotation, exprimée en tour par
balayés) sont égales. seconde.

CHAPITRE 15 - Le mouvement circulaire • 47


b. Scie circulaire : b. Fréquence de rotation de la poulie :
1 500 n = 570 ÷ 60 = 9,5 tr/s.
v = π.D.n = π.0, 40 × ≈ 31 m/s à l’unité près.
60 c. Le volant (1) étant solidaire de la poulie menante,
il a la même fréquence de rotation que celle-ci : 9,5
c. Pale d’hélicoptère :
tr/s. Le volant (2) entraîné directement et de même
v 157 diamètre que le volant (1) a aussi une fréquence de
n= = ≈ 5, 0 tr/s à 0,1 près.
π.D π × 10 rotation de 9,5 tr/s.
J’applique le cours d. La vitesse du ruban de la scie est :
v = π.D.n = π × 0,4 × 9,5 = 11,9 m/s.
5. Calculer une vitesse de coupe
a. Fréquence de rotation : n = 1 200 ÷ 60 = 20 tr/s. 9. Tapis roulant
b. D = 40 mm = 0,04 m. a. Vitesse de déplacement de la boîte, donc de la
c. La vitesse du tranchant de l’outil est : bande :
v = π.D.n = π × 0,04 × 20 = 2,5 m/s à 0,1 près. d 1,8
v= = = 0,15 m/s.
t 12
6. Le treuil b. Fréquence de rotation du rouleau :
a. D = 30 cm = 0,3 m. v 0,15
b. Fréquence de rotation : n = 36 ÷ 60 = 0,6 tr/s. n= = ≈ 0,19 tr/s
π.D π × 0, 25
c. Vitesse de remontée de la charge :
v = π.D.n = π × 0,3 × 0,6 = 0,57 m/s. soit 0,19 × 60 = 11 tr/min à l’unité près.
d. Durée de la remontée : 10. L’étiquetage des bouteilles
d 10 a. En 1 minute, le nombre de bouteilles traitées est
t= = ≈ 17,5 s.
v 0,57 de : 3 240 ÷ 60 = 54.
7. La bonne vitesse de coupe b. Le nombre de tours effectués par le carrousel en
a. D = 60 cm = 0,6 m. 1 minute est donc :
b. Fréquence de rotation : 54 ÷ 12 = 4,5 tr.
v 30 c. Fréquence de rotation du carrousel : n = 4,5 tr/min
n= = ≈ 16 tr/s. soit :
π.D π × 0, 6
4,5 ÷ 60 = 0,075 tr/s.
c. Conversion : n = 16 tr/s = 960 tr/min.
d. Dans le carrousel, la vitesse de déplacement d’une
8. Scie à ruban bouteille est donnée par :
a. D = 400 mm = 0,4 m. v = π .D.n = π × 0,6 × 0,075 = 0,14 m/s soit 14 cm/s.

LE DOCUMENT
■■La transmission du mouvement circulaire
• Réponse aux questions
1. Une moto peut utiliser une transmission pignons/chaîne. Certaines motos sont pourvues de
transmissions pignons/pignons (cardan) ou de transmissions pignons/courroie.
2. Le rapport de transmission de la bicyclette est :
Z1 42
= = 3.
Z2 14

3. À 1 tour de pédalier correspondent 3 tours effectués par la roue arrière.


4. Si la fréquence de rotation du pédalier est de 90 tr/min, celle de la roue arrière est de :
3 × 90 = 270 tr/min.
5. Pour un pignon de 23 dents entraîné par le plateau de 42 dents, le rapport de transmission est :
Z1 42
= = 1,826.
Z2 23
La fréquence de rotation de la roue est alors de :
1,826 × 90 ≈ 164 tr/min.

48 • CHAPITRE 15 - Le mouvement circulaire


PRÉPARER LE CCF
Vitesse moyenne et fréquence de rotation
Le tableau ci-dessous propose un barème d’évaluation pour aider à la notation de chaque élève.
Des fiches analogues, directement exploitables par l’enseignant, peuvent être téléchargées
sur le site de Hachette Éducation.

Vérifications Exploitation des résultats expérimentaux

1. Vérification du montage................................. 2 pts Calculs des vitesses moyennes


(4e ligne du tableau) . ............................................................. 1 pt
2. Réalisation et vérification
des premières mesures de distance Calculs des fréquences de rotation
et de durée .............................................................. 2 pts (5e ligne du tableau) . ............................................................. 1 pt
Réalisation et vérification Calculs des valeurs π . D . n
des secondes mesures de distance et vérification de l’égalité v = π . D . n . ......................... 1 pt
et de durée .............................................................. 2 pts
Remise en état du poste de travail ............. 1 pt

TOTAL ....................................................................... 7 pts TOTAL .............................................................................. 3 pts

CHAPITRE 15 - Le mouvement circulaire • 49


16 Actions mécaniques-Forces
Les activités
■■Activité 1 Quels sont les différents types d’actions mécaniques ?
• Matériel
Documents présentant différents types d’actions mécaniques.
Un aimant.
Une bille en acier.

• L’expérience
L’important est de bien différencier l’objet qui exerce l’action de celui qui la subit.

• Réponses aux questions


1. Lors du tir du penalty, l’action du pied sur le ballon provoque la mise en mouvement du ballon.
2. L’objet qui exerce l’action est en contact avec celui qui la subit dans le cas du tir du penalty,
(le pied au contact du ballon) du bateau (le vent est en contact avec la voile) et du perchiste
(les mains du perchiste en contact avec la perche).
3. Lors de l’attraction de la bille par l’aimant la bille n’est pas en contact avec l’aimant.
4. La surface de contact entre le pied et le ballon n’est pas aussi importante qu’entre le vent
et la voile. La première peut être considérée comme une action ponctuelle, la seconde comme
une action répartie.
5. L’action des mains sur la perche provoque la déformation de la perche.

■■Activité 2 Comment représenter graphiquement une force ?


• Matériel
Un tableau magnétique.
Des dynamomètres circulaires magnétiques.
Des dynamomètres linéaires.

• L’expérience
Les caractéristiques de la force correspondent à celle d’un segment fléché qui permet ainsi de la
représenter.

• Réponses aux questions


1. Tableau des caractéristiques de la force :
Force Point Droite Sens Valeur
d’application d’action
t
F main/fil O Fil 0,075 N

2. L’origine du segment fléché est le point O ;


– la direction du segment fléché est celle du fil ;
– le sens est vers le bas ;
0,075
– la longueur est = 7,5 cm.
0,01

50 • CHAPITRE 16 - Actions mécaniques-Forces


Les exercices
Je sais b. À gauche la valeur de la force est 2 N, à droite elle
est de 6,2 N.
1. Q.C.M.
1 : B et C ; 2 : A et B ; 3 : A, B et C ; 4 : C ; 5 : B et C. 8. Dresser le tableau des caractéristiques
d’une force
2. Les actions mécaniques de contact a. Le point d’application de la force est le point O ;
a. Les deux types d’action mécaniques de contact
b. sa droite d’action est horizontale ;
sont les actions ponctuelles et les actions réparties.
c. son sens est vers la droite ;
b. L’action de la pointe du compas sur la feuille est
d. En fonction de l’échelle inscrite sur le schéma, la
une action ponctuelle ; celle du vent sur la voile est
valeur de la force est 1,5 N.
une action répartie.
e. Tableau des caractéristiques  :
3. Les actions à distance Force Point Droite Sens Valeur
a. Les actions magnétiques sont un exemple d’action d’application d’action
à distance. t
b. Cette action est qualifiée d’ « action à distance », F main/crochet O Horizontale 1,5 N
car elle s’exerce sans qu’il y ait contact.

4. Mesure de la valeur des forces 9. Représenter une force


a. L’appareil servant à mesurer la valeur des for- O
ces s’appelle un dynamomètre.
b. L’unité légale de force  est le newton. Son sym-
bole est N.

J’applique
5. Distinguer différents types d’actions
mécaniques

Action Type
Effets
mécanique d’action

Le joueur Action de Modification


renvoie la balle contact localisée du mouvement
Fmarteau/clou
de tennis
Le footballeur Action de Mise en 10. Dresser le tableau des caractéristiques et
tire un penalty contact localisée mouvement représenter la force.
L’aimant attire Action à Attraction a. Tableau des caractéristiques de la force exercée
le clou distance répartie par la raquette sur la balle.

Le perchiste Action de Déformation Force Point Droite Sens Valeur


exerce contact localisée d’application d’action
une action sur t
F raquette/balle O Horizontale 1 000 N
la perche

6. Les effets d’une action mécanique


a. Les trois effets d’une action mécanique  sont la b. Représentation de
mise en mouvement d’un objet, la modification du la force à l’échelle 1 cm
mouvement ou la déformation. pour 500 N.
b. L’action du club sur la balle provoque la mise en
Fraquette/balle
O
mouvement de la balle.
Lors du crash-test, il se produit une déformation de
la voiture.

7. Utiliser correctement le dynamomètre


a. Les dynamomètres sont gradués en newton (N).

CHAPITRE 16 - Actions mécaniques-Forces • 51


11. La chargeuse 12. La punaise
t a. et b. mur
a. Tableau des caractéristiques de la force F
Force Point Droite Sens Valeur
d’application d’action Fpunaise/mur
t
F chargeuse/rocher O Verticale 25 000 N

b. Représentation Fchargeuse/roche
t
de la force F
à l’échelle 1 cm
pour 5 000 N :

LE DOCUMENT
■■Freinage et sécurité
• Réponses aux questions
1. La distance de freinage d’un véhicule dépend de la vitesse du véhicule, de l’état du véhicule
et de l’état de la chaussée.
2. Lorsqu’un conducteur est sous l’emprise de l’alcool ou de stupéfiants, son état influe
sur la distance de réaction.
3. La distance de freinage est augmentée si la route est mouillée.

PRÉPARER LE CCF

Les forces de frottement


Le tableau ci-dessous propose un barème d’évaluation pour aider à la notation de chaque élève.
Des fiches analogues, directement exploitables par l’enseignant, peuvent être téléchargées
sur le site de Hachette Éducation.

Vérifications Exploitation des résultats expérimentaux

1. Vérification des branchements La valeur de la force de frottement dépend


(résistance et DEL verte)................................. 2 pts de la masse du solide ........................................................ 0,5 pt
La valeur de la force de frottement augmente
2. Influence de la masse du solide
avec la masse du solide . ...................................................... 1 pt
(vérification des résultats) ............................ 2 pts
La valeur de la force de frottement dépend
3. Influence de la nature des surfaces de la nature des surfaces en contact . ....................... 0,5 pt
de contact (vérification des résultats). ... 2 pts La valeur de la force de frottement est la plus
Remise en état du poste de travail ............. 1 pt importante avec le caoutchouc ...................................... 1 pt

TOTAL ....................................................................... 7 pts TOTAL .............................................................................. 3 pts

52 • CHAPITRE 16 - Actions mécaniques-Forces


17 Équilibre d’un solide soumis à deux forces
Les activités
■■Activité 1 Quelles sont les conditions d’équilibre d’un solide soumis à
deux forces ?
• Matériel
Un tableau magnétique.
Deux dynamomètres circulaires magnétiques.
Une plaque de polystyrène.

• L’expérience
L’expérience avec le tableau magnétique est réalisée par le professeur, mais on peut distribuer à
chaque groupe d’élèves une plaque de polystyrène munie de deux ficelles aux extrémités desquelles
seront attachés deux dynamomètres. Le montage peut être représenté sur une feuille de papier afin de
décalquer la plaque et tracer la direction des deux ficelles.

• Réponse aux questions


À l’équilibre :
t t
1. les droites d’action de F 1 et F 2 sont confondues ;
t t
2. les forces F 1 et F 2 ont des sens opposés ;
t t
3. les forces F 1 et F 2 ont même valeur.

■■Activité 2 Différencier poids et masse d’un corps


• Matériel
Un dynamomètre circulaire.
Des objets : orange, stylos, trousses….
Une balance électronique.

• L’expérience
Les forces opposées vues dans l’activité 1 permettent d’expliquer l’existence des deux forces
appliquées à un solide en équilibre : le poids du solide et la force exercée par le dynamomètre.
La confusion entre poids et masse est fréquente. On montrera que chaque grandeur se mesure avec
un appareil différent. (Même si la balance électronique est en fait un dynamomètre gradué en unités
de masse).

• Réponse à la question
P
Le rapport ne dépend pas de l’objet, il ne dépend que du lieu.
m

CHAPITRE 17 - Équilibre d’un solide soumis à deux forces • 53


Les exercices

Je sais b.

1. Q.C.M.
1 : A et C ; 2 : A et C ; 3 : B ; 4 : B ; 5 : C ; 6 : C.

2. Quelques définitions
a. Le poids d’un objet est la force exercée par la Terre
sur cet objet. G
b. Le poids d’un objet se mesure en newton (N).
c. On mesure le poids d’un objet avec un dynamo-
mètre.

3. Conditions d’équilibre d’un objet


t t
Si un objet soumis à deux forces F 1 et F 2 est en équi-
libre, alors les deux forces ont même droite d’action,
même valeur mais des sens opposés.

4. Une formule importante


La relation qui existe entre le poids et la masse d’un
corps est P = m.g.
a. La lettre P représente le poids du corps.
On l’exprime en newton (N). P
b. La lettre m représente la masse du corps.
On l’exprime en kilogramme (kg). 10. Étude de la plaque de charge d’un chariot
c. La lettre g représente l’intensité de la pesanteur. élévateur
On l’exprime en newton par kilogramme (N/kg). a. 6 300 mm = 6,3 m ; 600 mm = 0,6 m.
b. La hauteur maximale d’élévation pour une charge
5. Direction ou sens ? de 1 125 kg dont le centre de gravité est situé à 800 mm
a. une droite d’action définit une direction ; du bord du mât est 4 900 mm = 4,9 m.
b. une flèche indique un sens ; c. La hauteur maximale d’élévation pour une charge
c. le mot « vertical » indique une direction. de 950 kg dont le centre de gravité est situé à 800 mm
6. À la poste du bord du mât est 6 300 mm = 6,3 m.
La postière indique à la cliente la masse de la lettre 11. Équilibre d’une bille
puisqu’elle donne la valeur en gramme. a. Les actions mécaniques qui s’exercent sur la bille
t t
sont le poids P de la bille et la tension T du fil.
t
J’applique b. Le poids P est une action à distance répartie. La
t
7. Valeur des forces à l’équilibre tension T du fil est une action de contact localisée.
a. La valeur de la force exercée par le fil sur le c. La masse de la bille est :
lustre est de 5 N. P 5
m= = = 0,5 kg.
b. La valeur de la force exercée par la table sur le g 10
livre est de 3 N. d. Le tableau des caractéristiques des forces est :

8. Erreur d’unité
Force Point Droite Sens Valeur
On devrait écrire « masse nette » 370 g.
d’application d’action
9. La représentation du poids
Le poids du ballon s’applique au point G, sa droite t
Poids P O 5 N
d’action est la verticale, son sens est vers le bas et sa
valeur est de 5 N.
Tension du B, point
t d’attache de la 5 N
câble T bille et du fil

54 • CHAPITRE 17 - Équilibre d’un solide soumis à deux forces


12. Les élingues.
a. Le poids P de la poutre est P = m . g = 300 × 10 = 3 000 N.
b. Tableau des caractéristiques des trois forces qui s’exercent sur la poutre :
Point Droite
Force Sens Valeur
d’application d’action

t G 3 000 N
Poids P

Tension
t A 30° 3 000 N
T1
du câble
AC

Tension
t B 30° 3000 N
T2 du
câble BC

c. Représentation des trois forces

T1 T2

30° 30°
A B

LE DOCUMENT
■■Le poids d’un objet n’est pas le même sur la Terre et sur la Lune
Réponses aux questions
1. Le savant qui est à l’origine de la gravitation universelle est Isaac Newton. Il était de
nationalité anglaise.
2. La masse de l’objet sur Terre est 5 kg. Son poids est 50 N.
3. La masse de l’objet sur la Lune est 5 kg. Son poids est 8 N.
4. La masse est la grandeur qui n’a pas varié entre la Terre et la Lune. C’est le poids qui a varié.
Le poids de l’objet est environ 6 fois plus petit sur la Lune que sur la Terre.

CHAPITRE 17 - Équilibre d’un solide soumis à deux forces • 55


PRÉPARER LE CCF

Le centre de gravité
Le tableau ci-dessous propose un barème d’évaluation pour aider à la notation de chaque élève.
Des fiches analogues, directement exploitables par l’enseignant, peuvent être téléchargées
sur le site de Hachette Éducation.

Vérifications Exploitation des résultats expérimentaux

1. Vérification du tracé de la verticale Le centre de gravité d’une plaque rectangulaire


passant par A.......................................................... 2 pts se trouve à l’intersection des axes de symétrie
du rectangle. (C’est aussi le point d’intersection
2. Vérification du tracé des verticales
des diagonales)........................................................................... 1 pt
passant par B et C ............................................... 2 pts
Le centre de gravité d’une plaque en forme
3. Vérification de la position
de disque se trouve au centre du disque . ............. 0,5 pt
du centre de gravité. .......................................... 2 pts
Remise en état du poste de travail ............. 1 pt Celui d’une plaque en forme de triangle
équilatéral se trouve à l’intersection
des médianes.............................................................................. 1 pt

Celui d’une boule se trouve au centre


de la boule. .............................................................................. 0,5 pt

TOTAL ....................................................................... 7 pts TOTAL .............................................................................. 3 pts

56 • CHAPITRE 17 - Moment d’une force - Moment d’un couple de forces


18 Moment d’une force
Moment d’un couple de forces

Les activités
■■Activité 1 Que désigne le « moment d’une force »
par rapport à un axe de rotation ?
• Matériel
Un tableau magnétique.
Un dynamomètre circulaire.
Une barre à trous.
Index, ressort de rappel, plot magnétique.

• L’expérience
Indiquer que la barre à trous restant toujours horizontale, les effets des différentes forces sont
équivalents.

• Réponse aux questions


1. La force appliquée est d’autant plus faible que l’on s’éloigne de l’axe de rotation O.
2. Les effets de la force sont caractérisés par le produit F . d.

■■Activité 2 Comment calculer le moment d’un couple ?


• Matériel
Un tableau magnétique.
Deux dynamomètres circulaires magnétiques.
Une barre à trous.
Index, ressort de rappel, plot magnétique.

• L’expérience
Puisque la barre reste toujours horizontale, les effets des différents couples de forces sont équivalents.

• Réponses aux questions


1. Dans le cas du document 4 page 121, la barre a tendance à tourner dans le sens des aiguilles
d’une montre, sous l’effet du couple de forces.
2. Puisque la barre reste toujours horizontale, les effets des différents couples de forces sont
équivalents.
3. Lorsque d augmente, la valeur F des forces diminue.
4. L’effet d’un couple de forces est caractérisé par le produit F . d.

CHAPITRE 18 - Moment d’une force - Moment d’un couple de forces • 57


Les exercices

Je sais 8. Le tournevis
a. d = 0,010 mètre.
1. Q.C.M. b. Le moment du couple exercé sur la tête de la vis
1 : B ; 2 : C ; 3 : C ; 4 : B ; 5 : C. est :
2. Moment d’une force ℳC = F . d = 20 × 0,010 = 0,20 N.m.
a. La lettre F représente la valeur de la force. On doit 9. La brouette
l’exprimer en newton (N). t
b. La lettre d représente le bras de levier, c’est-à-dire a. La valeur de la force F exercée par l’utilisateur est
la distance entre la droite d’action de la force et l’axe inférieure au poids du système brouette-terre.
t
de rotation. On doit l’exprimer en mètre (m). b. La distance de la droite d’action de P à l’axe de
rotation O est d1 = 0,4 m.
3. L’effet du bras de levier t
Il vaut mieux choisir la clé de longueur 250 mm, car la La distance de la droite d’action de F à l’axe de rota-
force à exercer sera plus faible pour desserrer l’écrou tion O est
étant donné que le bras de levier sera plus impor- d2 = 0,4 + 0,8 m = 1,2 m.
tant. t
c. Le moment du poids P par rapport au point O est
4. Moment d’un couple ℳt
P/O = 600 × 0,4 = 240 N.m.
a. La lettre F représente la valeur commune des for- t
d. Le moment de la force F par rapport au point
t t
ces F 1 et F 2 et la lettre d représente la distance entre O est
leurs droites d’action.
ℳt
F/O = 200 × 1,2 = 240 N.m.
b. F doit être exprimé en newton (N) et d  doit être e. Ces deux moments sont égaux.
exprimé en mètre (m).
c. Le moment ℳC  est exprimé en newton-mètre 10. Le pied-de-biche
(N.m) t
a. Le moment de la force F par rapport au point O
est :
J’applique F.OA = 200 × 0,5 = 100 N.m.
5. Relever le skate t
b. Le moment de la force F est égal à
t F’.OB = F’ × 0,05 N.m.
a. Le moment de la force F est donné par la relation
ℳ = F . d. Puisque les deux moments sont égaux :
F est exprimée en newton (N), d, en mètre (m) et ℳ en 100
F’ × 0,05 = 100, d’où F’ =  = 2 000 N.
newton-mètre (N.m). 0,05
b. Le moment de la force exercé par le pied de Frédé-
ric est 11. La clé à choc pneumatique
F . d = 10 × 0,2 = 2 N.m. La valeur des forces constituant ce couple est
6. La clé à pipe 217
F’ =  = 8 680 N.
a. d = 0,20 m. 0,025
t 12. Soulever une charge
b. Le moment de la force F par rapport à O est égal
à :
t
a. Le moment du poids P par rapport à l’axe de rota-
ℳt tion est
F/O = F . d = 100 × 0,2 = 20 N.m.
P.d = 300 × 0,4 = 120 N.m.
7. Prévoir le sens de rotation d’un solide soumis
à un couple de forces b. Consignes de sécurité pour soulever une charge.
a. Le sens de rotation de la clé est celui des aiguilles – Plier les genoux et soulever la charge avec le dos
d’une montre. droit.
b. Si le sens de rotation de la clé était inversé, le cou- – Ne pas hésiter à utiliser un moyen de manutention
ple de forces aurait pour effet de dévisser l’écrou. si la charge est lourde. Se faire aider si nécessaire.

58 • CHAPITRE 18 - Moment d’une force - Moment d’un couple de forces


LE DOCUMENT

■■Le couple moteur dans une automobile


• Réponses aux questions
1. Sur l’axe horizontal, la fréquence de rotation du moteur est donnée en tour par minute (tr/min).
2. Sur l’axe vertical de gauche, on porte les valeurs de la puissance du moteur en kW. Sur l’axe vertical
de droite, on porte les valeurs du couple moteur en N.m.
3. Le couple passe par un maximum égal à 190 N.m.
4. Le couple est maximum pour un régime du moteur compris entre 2 200 et 3 600 N.m.

PRÉPARER LE CCF

Le moment d’une force


Le tableau ci-dessous propose un barème d’évaluation pour aider à la notation de chaque élève.
Des fiches analogues, directement exploitables par l’enseignant, peuvent être téléchargées
sur le site de Hachette Éducation.

Vérifications Exploitation des résultats expérimentaux

1. Vérification de la valeur de P...................... 2 pts

2. Vérification du montage ................................ 2 pts Mesures et calculs : tableau complété . .................... 4 pts
Remise en état du poste de travail ............. 1 pt
Les moments des deux forces sont égaux ............ 0,5 pt

Ce résultat s’explique par le fait


que la barre reste en équilibre . ................................... 0,5 pt

TOTAL ....................................................................... 5 pts TOTAL .............................................................................. 5 pts

CHAPITRE 18 - Moment d’une force Moment d’un couple de forces • 59


19 Les sons
Les activités
■■Activité 1 Comment identifier un son ?
• Matériel
- Diapason.
- Instrument de musique.
- Oscilloscope ou système d’acquisition EXAO.
- Microphone.

• L’expérience
L’idéal est de réaliser cette expérience avec un système d’acquisition exao relié à un vidéoprojecteur.
Les courbes sont projetées au mur, donc très visibles, et le système exao permet de mesurer
simplement la période. Ce type de système permet aussi d’afficher toutes les courbes sur le même
écran afin de les comparer plus facilement.
Cette manipulation peut être réalisée par des élèves guidés par le professeur.

• Réponse aux questions


1. Les fréquences sont identiques mais les formes sont différentes.
2. C’est la forme des courbes qui permet de différencier les sons émis.
Cette caractéristique est appelée le timbre de l’instrument.

■■Activité 2 Comment évolue le niveau d’intensité sonore en décibel (dB) ?


• Matériel 
- Un générateur de Basse Fréquence.
- Un haut-parleur.
- Un multimètre.
- Un sonomètre.
- Des fils de connexion.

• L’expérience
Cette expérience doit être réalisée par un élève devant le reste de la classe ou par le professeur.

• Réponses aux questions


1. Lorsque l’amplitude de la tension augmente, le niveau d’intensité sonore augmente.
2. Lorsque l’on s’éloigne de l’émetteur, le niveau d’intensité sonore diminue.

Les exercices

Je sais 3. Période fréquence.


a. Oscillogramme 1 : T1 = 5 carreaux
1. Q.C.M. soit T1 = 5 × 0,1 × 10-3 = 0,5 × 10-3 seconde.
1 : A, C ; 2 : B ; 3 : A ; 4 : C ;
Oscillogramme 2 : T2 = 4 carreaux
5 : B, C ; 6 : A, B ; 7 : A, B, C.
soit T2 = 4 × 0,1 × 10-3 = 0,4 × 10-3 seconde.
2. Son ou bruit 1 1
b. Oscillogramme 1 : f1 = = = 2000 Hz.
Les oscillogrammes 1 et 2 représentent des sons purs, T1 0,5 ×10 −3
le 3 représente un son complexe et le 4 représente un 1 1
bruit. Oscillogramme 2 : f 2 = = = 2500 Hz.
T2 0,4 ×10 −3

60 • CHAPITRE 19 - Les sons


4. Mettre de l’ordre
J’applique
Fréquence
8. Sans calcul
Son 4 10Hz
a. Oui ce sont des sons purs, car les courbes sont des
Son 2 100 Hz sinusoïdes.
Son 5 1 kHz b. Par ordre croissant de fréquences des oscillations :
3, 1, 2, 4.
Son 3 10 000 Hz
c. L’oscillogramme du son le plus aigu est le 4.
Son 1 100 kHz
9. L’abus de décibels est dangereux
5. Comment positionner un sonomètre ? a. Dans un restaurant : L = 70 dB ;
a. Un sonomètre mesure le niveau d’intensité lors d’un concert de rock : L = 115 dB.
sonore. b. La durée maximale d’écoute d’un baladeur à
b. Son unité de mesure est le décibel. pleine puissance sans risque pour l’audition est de
c. La bonne position du sonomètre est la plus à 20 minutes.
droite. Le microphone du sonomètre doit être dirigé c. Il est dangereux pour l’audition de :
vers l’émetteur dont on veut mesurer le niveau d’in- – tirer au fusil ;
tensité sonore. – tondre le gazon ;
– travailler dans un atelier d’ajustage.
6. Isolants phoniques
d. Un son implique un « risque avéré » pour l’audition
a. Le verre permet d’avoir le niveau d’intensité
à partir de 100 décibels et 20 minutes d’écoute.
sonore le plus bas.
b. Classement des matériaux du plus isolant au 10. Lecture de documentation
moins isolant : a. La première ligne du tableau indique la fréquence
Verre, Liège, Bois, Air. donc la hauteur du son.
b. Ces bouchons sont plus efficaces pour les sons
7. S’éloigner de la source
aigus que pour les sons graves.
Au point A : L4 = 108 dB
c. Oui elle peut se protéger avec ces bouchons car à
Au point B : L2 = 98 dB
4000 hertz, l’affaiblissement est de 40,3 décibels. Elle
Au point C : L1 = 75 dB
percevra donc un son de :
Au point D : L3 = 64 dB
120 − 40,3 = 79,7 décibels.

LE DOCUMENT

■■Comment protéger son audition ?


• Réponses aux questions
1. C’est la cochlée et les cellules cillées qu’elle contient qui sont le plus sensibles aux forts niveaux
d’intensité sonore.
2. Non, on ne guérit pas systématiquement d’une perte d’audition.
3. Si à 2 mètres, L = 98 dB ; à 4 mètres, L = 92 dB ; à 8 mètres L = 86 dB ; à 16 mètre L = 80 dB.
Il faut donc se placer à au moins 16 mètres de cette source sonore pour être en sécurité.

CHAPITRE 19 - Les sons • 61


PRÉPARER LE CCF

Absorption et isolation acoustique


Le tableau ci-dessous propose un barème d’évaluation pour aider à la notation de chaque élève.
Des fiches analogues, directement exploitables par l’enseignant, peuvent être téléchargées
sur le site de Hachette Éducation.

Vérifications Exploitation des résultats expérimentaux

1. Montage....................................................................... 1 pt Réponses aux questions :


Réglage générateur ............................................... 1 pt – Effet des parois alvéolées ................................................ 1 pt
– Pouvoir isolant en fonction de la fréquence ....... 1 pt
2. Utilisation du sonomètre. ................................. 1 pt
– Matériau le plus isolant .................................................... 1 pt
Qualité des mesures . ........................................... 1 pt

3. Qualité des résultats.......................................... 2 pts

4. Remise en état du poste de travail ............. 1 pt

TOTAL ....................................................................... 7 pts TOTAL .............................................................................. 3 pts

20 Température et changement d’état


Les activités
■■Activité 1. Comment mesurer une température ?
• Matériel pour un groupe d’élèves
Un thermomètre numérique.
Un thermomètre à alcool.

• Réponse aux questions


1. Le thermomètre numérique indique 21,8 °C ; le thermomètre à alcool indique 22 °C.
La température s’exprime habituellement en degré Celsius (°C).
2. Effectuer des mesures en classe.
3. Correspondance : 22 °C ≈ 72 °F.

■■Activité 2. Quels sont les changements d’état de l’eau ?


Expérience 1
• Matériel
Un cristallisoir.
Un bécher.
Un thermomètre numérique.

• Produits
De l’eau, de la glace et du sel de cuisine.

• Réponse à la question
Ce changement d’état est la solidification de l’eau. Il se produit à 0 °C.

62 • CHAPITRE 20 - Température et changement d’état


Expérience 2
• Matériel
Un bécher.
Un thermomètre numérique.
Une coupelle.
Une pince en bois.
Une plaque chauffante.

• Produit
De l’eau.

• Réponse aux questions


1. Ce changement d’état observé est l’ébullition de l’eau. Il se produit à 100 °C.
2. Les gouttes d’eau proviennent de la condensation de la vapeur d’eau sur la coupelle froide.

Les exercices
Je sais J’applique
1. Q.C.M. 8. Reconnaître un changement d’état
1 : B et C ; 2 : B ; 3 : C ; 4 : A ; 5 : B. a. Solidification. b. Solidification.
c. Fusion. d. Condensation.
2. Lire une température
e. Vaporisation.
Températures : 1 → 46 °C ; 2 → 33,6 °C ; 3 → 16,4 °C.
9. Convertir
3. Degré Celsius et Fahrenheit
a. T = θ + 273. Pour la solidification de l’eau :
Températures : 1  24,0 °C ; 2  75,2 °F.
T = 0 + 273 = 273 K.
4. Degré Celsius et degré Fahrenheit b. Fusion du dihydrogène :
a. 0 °F correspond à – 18 °C. T = -259 + 273 = 14 K.
b. 0 °C correspond à 32 °F. c. Fusion du fer :
c. 25 °C correspond à 77 °F ; θ = T – 273 = 1 808 – 273 = 1 535 °C.
– 12 °F correspond à – 25 °C.
10. Tracer une courbe
5. Nommer les changements d’état a. Graphique ci-dessous :

Fusion Vaporisation θ (°C)


SOLIDE LIQUIDE GAZ

Solidification Condensation

20
6. Réchauffer, refroidir
a. Quand on chauffe un solide, il peut fondre
(fusion) ;
quand on chauffe un liquide, il peut se vaporiser
(vaporisation).
b. Quand on refroidit un liquide, il peut se solidifier 10
(solidification) ;
quand on refroidit un gaz, il peut se condenser
(condensation).

7. Refroidir de l’eau
La solidification de l’eau pure correspond au graphi- 2
t (s)
que 2 car on observe un palier de la température à 0
0 °C. 0 20 100 200

CHAPITRE 20 - Température et changement d’état • 63


b. La température de solidification du cyclohexane c. Analyse de la courbe (B) :
correspond à celle du palier observé : 6,5 °C. pour t = 60 s, la substance est à l’état liquide ;
à l’instant t = 140 s, elle est à l’état de mélange {liquide
11. Corps pur ou mélange
+ gaz}.
a. Ce sont des vaporisations car la température est
d. La température de vaporisation de l’eau du robi-
croissante et voisine de 100 °C.
net est de 100 °C.
b. Le tube 1 (eau du robinet) → Courbe (B) ;
le tube 2 (eau très salée) → Courbe (A).

LE DOCUMENT

■■Les cartes météorologiques


• Réponse aux questions
1. Sur la carte des isothermes de Juillet, les régions d’Afrique du Nord sont les plus chaudes.
2. 1 025 hPa est la valeur d’une forte pression correspondant à un anticyclone (zone de beau temps).
3. Vent d’ouest de 30 nœuds : Vent de sud-est de 15 nœuds :

PRÉPARER LE CCF

Changement d’état de la paraffine


Le tableau ci-dessous propose un barème d’évaluation pour aider à la notation de chaque élève.
Des fiches analogues, directement exploitables par l’enseignant, peuvent être téléchargées
sur le site de Hachette Éducation.

Vérifications Exploitation des résultats expérimentaux

1. Vérification du montage.................................... 1 pt Construction du graphique . ......................................... 1,5 pt


2. Vérification des mesures initiales Réponses aux questions :
(température et état de la paraffine) . ..... 2 pts – la température de changement d’état. ................ 0,5 pt
Vérification des mesures suivantes.......... 3 pts – l’évolution de la température . .................................. 0,5 pt
3. Remise en état du poste de travail ............. 1 pt – la nature de la paraffine (mélange) . ...................... 0,5 pt

TOTAL ....................................................................... 7 pts TOTAL .............................................................................. 3 pts

64 • CHAPITRE 20 - Température et changement d’état

Vous aimerez peut-être aussi